QUIZ COMPILATIONS – JULY PART 1

96
GOALTIDE IAS ACADEMY GOALTIDEIAS.COM | [email protected] QUIZ COMPILATIONS – JULY PART 1

Transcript of QUIZ COMPILATIONS – JULY PART 1

Page 1: QUIZ COMPILATIONS – JULY PART 1

GOALTIDE IAS ACADEMY GOALTIDEIAS.COM | [email protected]

QUIZ COMPILATIONS – JULY PART 1

Page 2: QUIZ COMPILATIONS – JULY PART 1

GOALTIDE IAS ACADEMY 1

QUIZ COMPILATIONS – JULY PART 1

Q1. Consider the following statements.

1. As pe the Members of Lok Sabha (Disqualification on the Grounds of Defection) Rules,

1985, a time limit has been imposed on Speaker to decide the defection case of the

House.

2. An independent member of Parliament is not disqualified if she/he joins any political

party after her/his election.

3. A member who is disqualified for being a member of the House under Tenth Schedule

shall also be disqualified to hold any post where the salary or remuneration for such

office is paid out of the public revenue of the Government of India.

Which of the above statements is/are correct?

a. 3 only

b. 2 and 3 only

c. 1 and 3 only

d. 1, 2 and 3

Solution: a

Explanation:

First statement is incorrect:

If you follow news, you can easily answer first statement correctly.

According to the "Members of Lok Sabha (Disqualification on the Grounds of Defection)

Rules, 1985" and "The Members of Rajya Sabha (Disqualification on the Ground of

Defection) Rules, 1985", a time limit has been imposed on the Leaders to bring the

defecting members to the notice of the Speaker whereas no such rules as to the time limit

for deciding the said matter has been mentioned.

This creates a situation where Speakers often use this discretionary power as a species of

'Pocket veto' and often lead to unwarranted delays in deciding matters of defection.

If you have enough time, and want to read rules in detail, click here.

Second statement is incorrect:

An independent member of Parliament or a State Legislature will also be disqualified if

she/he joins any political party after her/his election.

Third statement is correct:

Article 361B of the Constitution inter alia provides— “A member of a House belonging to

any political party who is disqualified for being a member of the House under paragraph 2 of

Page 3: QUIZ COMPILATIONS – JULY PART 1

GOALTIDE IAS ACADEMY 2

QUIZ COMPILATIONS – JULY PART 1

the Tenth Schedule shall also be disqualified to hold any remunerative political post for

duration of the period commencing from the date of her/his disqualification till the date on

which the term of her/his office as such member would expire or till the date on which

she/he contests an election to a House and is declared elected, whichever is earlier.”

The expression “remunerative political post” means any office under the Government of

India or the Government of a State where the salary or remuneration for such office is paid

out of the public revenue of the Government of India or the Government of the State, as the

case may be.

Q2. A person to be qualified as a Judicial Member of Central Administrative Tribunal if he is:

1. has been, a Judge of a High Court

2. has, for one year, held the post of Secretary to the Government of India in the

Department of Legal Affairs

3. Advocate with ten years’ experience as Judicial Member only in High Court.

Select the correct code.

a. 1 and 2 only

b. 2 and 3 only

c. 1 and 3 only

d. 1, 2 and 3

Solution: a

Explanation:

A difficult question. Reason is:

The Central Government has amended the Tribunal, Appellate Tribunal and other

Authorities (Qualifications, Experience and other Conditions of Service of Members) Rules,

2020 enabling appointment of Advocates with ten years’ experience as Judicial Members

of various Tribunals (not only High Court).

What was before amendment?

After amendment:

A person shall not be qualified for appointment,– (a) as a Judicial Member, unless he,– (i) is,

or has been, a Judge of a High Court; or (ii) has, for one year, held the post of Secretary to

the Government of India in the Department of Legal Affairs or the Legislative Department

including Member –Secretary, Law Commission of India; or (iii) has, for two years, held a

post of Additional Secretary to the Government of India in the Department of Legal Affairs

Page 4: QUIZ COMPILATIONS – JULY PART 1

GOALTIDE IAS ACADEMY 3

QUIZ COMPILATIONS – JULY PART 1

or Legislative Department; or (iv) has, for a combined period of ten years, been a District

Judge and Additional District Judge; or (v) has, for ten years, been an advocate with

substantial experience in litigation in service matters in Central Administrative Tribunal,

Armed Forces Tribunal, High Court or Supreme Court.

So, now for every such tribunal, for say, National Green Tribunal: It will read as:

(a) is, or has been, a Judge of a High Court; or (b) has, for a combined period of ten

years, been a District Judge and Additional District Judge; or (c)has, for ten years,

been an advocate with substantial experience in litigation in matters relating to

environment and forest in National Green Tribunal, High Court or Supreme Court.

Q3. With respect to “Sea snot” recently observed in Turkey, consider the following

statements.

1. It is formed when algae are overloaded with nutrients as a result of water pollution

and climate change.

2. It was observed in Dead Sea.

Which of the above statement sis/are correct?

a. 1 only

b. 2 only

c. Both 1 and 2

d. Neither 1 nor 2

Solution: a

Explanation:

First statement is correct:

When algae are overloaded with nutrients as a result of water pollution and climate change,

it begins to create a thick slimy layer on the surface of the water. This layer is colloquially

known as sea snot.

It was first discovered in Turkey in 2007. Also known as ‘marine mucilage’, it is a thick, slimy

grey-brown sheet that is formed by dead and living organic material.

Algae are the prime source of oxygen in water bodies. However, if their growth is

overlooked, they could lead to the formation of this mucilage that could block sunlight

from entering the deep waters of the oceans.

Some of the sea snot has sunk below the water surface, suffocating the seabed. The

proliferation of the organic matter can also flourish when nutrient-rich sewage flows into

seawater as it contains a wide variety of micro-organisms.

Second statement is incorrect: It is Sea of Marmara.

Page 5: QUIZ COMPILATIONS – JULY PART 1

GOALTIDE IAS ACADEMY 4

QUIZ COMPILATIONS – JULY PART 1

Q4. “Rabari and Maldhari” tribes belong to:

a. North West India

b. North East Region

c. Western Ghats

d. Himalayas

Solution: a

Explanation:

They are tribes of North West India.

Q5. Consider the following.

1. Borneo Island

2. Sumatra Island

3. Sulawesi Island

Arrange the above islands from West to East.

a. 2-1-3

b. 2-3-1

c. 1-2-3

d. 1-3-2

Solution: a

Explanation:

The correct sequence is:

Sumatra Island-Borneo-Sulawesi

Page 6: QUIZ COMPILATIONS – JULY PART 1

GOALTIDE IAS ACADEMY 5

QUIZ COMPILATIONS – JULY PART 1

Also see other places in the map. No need to run.

Q6. Which of the following correctly defines Molecular ecology?

a. Reducing the environmental impact of formulas and packaging and using more and

more renewable raw materials that are sustainably sourced

b. use of natural substances that include enzymes from biological sources or whole cells

to speed up chemical reactions.

c. biocultural approach to human health that integrates perspectives from

anthropological political economy, ecology, and human adaptability.

d. It is a hybrid field that combines molecular biology techniques with ecological data to

make sense of natural processes such as the growth or decline of populations,

formation of new species, extinctions.

Solution: d

Explanation:

As urbanisation, deforestation, loss of wildlife, and human-wildlife conflicts continue to

spiral up, there is a need to use every available tool available, to help protect what is left of

the natural world. Molecular ecology is one such tool for conservation and can help in

wildlife disease management and forensics in illegal trade.

What is molecular ecology?

Molecular ecology is a hybrid field that combines molecular biology techniques with

ecological data to make sense of natural processes such as the growth or decline of

populations, formation of new species, extinctions and invasiveness.

Molecular ecology is used to estimate population genetic diversities to aid wildlife breeding

and conservation efforts, define species for conservation policy, track diseases, and combat

poaching.

Q7. Consider the following statements about the Parliamentary privileges in India:

Page 7: QUIZ COMPILATIONS – JULY PART 1

GOALTIDE IAS ACADEMY 6

QUIZ COMPILATIONS – JULY PART 1

1. President of India, who is integral part of the Indian Parliament, also enjoys these

privileges.

2. Normally, a Member of Parliament can raise a question involving a breach of

privilege in the Houses of Parliament even without the consent of the Presiding

Officer of that House.

3. The Parliament of India has not codified its privileges.

Which of the statements given above is/are correct?

a. 3 only

b. 2 and 3 only

c. 1 and 2 only

d. 1, 2 and 3

Solution: a

Explanation:

Parliamentary privilege refers to rights and immunities enjoyed by Parliament as an

institution and MPs in their individual capacity, without which they cannot discharge their

functions as entrusted upon them by the Constitution.

President, though an integral part of the parliament, does not enjoy parliamentary

privileges. These rights are mainly from the members of both the Houses of Parliament.

Apart from this, these rights are also given to those individuals who speak and participate in

any committee of the Parliament, which includes the Attorney General of India and the

Union Ministers. So, statement 1 is not correct.

A member can raise a question involving a breach of privilege only with the consent of the

Speaker or the Chairperson. So, statement 2 is not correct.

The Parliament has not yet codified its privileges. According to the Constitution, the powers,

privileges and immunities of Parliament and MP's are to be defined by Parliament. No law

has so far been enacted in this respect. So, statement 3 is correct.

Q8. In the context of Ancient Indian sculptures, The Ardhaparyanka asana and Achamana

Mudra are associated with which of these popular Indian deities?

a. Krishna

b. Shiva

c. Vishnu

d. Brahma

Solution: b

Explanation:

The bronze casting technique and making of bronze images of traditional icons reached a

high stage of development in south India during the medieval period.

Page 8: QUIZ COMPILATIONS – JULY PART 1

GOALTIDE IAS ACADEMY 7

QUIZ COMPILATIONS – JULY PART 1

Among the Pallava period bronze of the 8th century, the best one is the icon of Shiva

seated in Ardhaparyanka asana (one leg kept dangling). The right hand is in the Achamana

Mudra gesture, suggesting that he is about to drink poison.

Q9. Consider the following statements regarding Wholesale Price Index.

1. WPI basket does not cover services.

2. The weightage of Fuel and Power is more than manufactured products in WPI

basket.

3. Currently, Indirect taxes are not part of WPI calculation in India.

Which of the above statements is/are correct?

a. 1 only

b. 2 and 3 only

c. 1 and 3 only

d. 1, 2 and 3

Solution: c

Explanation:

So, we will go to WPI original website and gather all information about it.

First statement is correct.

Second statement is incorrect:

Page 9: QUIZ COMPILATIONS – JULY PART 1

GOALTIDE IAS ACADEMY 8

QUIZ COMPILATIONS – JULY PART 1

Third statement is correct:

We are pasting link of all WPI information. You can refer it for any other information.

Link: https://eaindustry.nic.in/uploaded_files/FAQs_on_WPI.pdf

Page 10: QUIZ COMPILATIONS – JULY PART 1

GOALTIDE IAS ACADEMY 9

QUIZ COMPILATIONS – JULY PART 1

Q10. In the context of India History, “Tabernacle of New Dispensation” founded by:

a. Raja Ram Mohan Roy

b. Rabindra Nath Tagore

c. Keshub Chandra Sen

d. Birendra “Thomas” Roy.

Solution: c

Explanation:

“Tabernacle of New Dispensation” already has been asked by UPSC indirectly in options.

You should know if appearing for October 4 exam. We have also covered in our Test Series.

In 1868, Keshub laid the foundation stone of his new church, the Tabernacle of New

Dispensation.

Q11. Safeguarding constitutionalism lies with:

a. Judiciary

b. Executive

c. Legislative

d. All the above

Solution: d

Explanation:

Now, this question needs interpretation. Everyone will give their own answers.

We will use here interpretation of CJI Ramana, who opined that the importance of the

Judiciary should not blind us to the fact that the responsibility of safeguarding

constitutionalism, lies not just on the Courts, and all the three organs of the State, i.e., the

executive, legislature and the Judiciary, are equal repositories of Constitutional trust.

We have mentioned this in our 30 June CA. Hope you are reading them.

Q12. Consider the following statements regarding Right of Children to Free and Compulsory

Education Act, 2009 (RTE Act).

1. To improve performances of children in schools, Continuous Comprehensive Evaluation

(CCE) system was introduced as per the Act, to ensure grade appropriate learning

outcomes in schools.

2. In India, the right to education was made a fundamental right by inserting Article 21A

by the Constitution (Fourty-second Amendment) Act, 1976.

3. Pre School education is excluded from the ambit of RTE 2009.

Which of the above statements is/are correct?

a. 1 and 2 only

b. 1 and 3 only

Page 11: QUIZ COMPILATIONS – JULY PART 1

GOALTIDE IAS ACADEMY 10

QUIZ COMPILATIONS – JULY PART 1

c. 3 only

d. 1 only

Solution: b

Explanation:

First statement is correct.

Continuous and Comprehensive Evaluation, commonly known as CCE, is introduced as

school-based system of evaluation as per Right of Children to Free and Compulsory

Education Act–2009 (RTE Act–2009), which came into force since April 2010. The Act

requires that CCE be implemented for children till the completion of elementary schooling.

In view of the reduced predominance or prohibition of external examination up to the

elementary level, it is all the more important for different stakeholders in school education,

especially teachers, to understand and use CCE meaningfully to enable each child learn and

progress.

Second statement is incorrect:

Its Eighty-sixth Amendment Act, 2002. The Constitution (Eighty-sixth Amendment) Act,

2002 inserted Article 21-A in the Constitution of India to provide free and compulsory

education of all children in the age group of six to fourteen years as a Fundamental Right in

such a manner as the State may, by law, determine.

Third statement is correct:

RTE Act, 2009, excluded preschool education from its ambit.

The RTE Act should cover children in the 0-6 age group and ensure they get care, education

and development. But the Central and state governments are cutting their education

budgets, and abdicating their duty and passing it on to for-profit private education

providers.

Q13. Consider the following statements.

1. E-waste rules doesn’t apply to MSMEs.

2. Producer Responsibility Organisation under E-waste Rules, 2016 supervise the

function producers under Extended Producer Responsibility (EPR).

3. More than 90% of e-waste in India is recycled by the informal sector.

Which of the above statements is/are correct?

a. 1 and 2 only

b. 3 only

c. 1 and 3 only

d. 1, 2 and 3

Solution: c

Explanation:

Page 12: QUIZ COMPILATIONS – JULY PART 1

GOALTIDE IAS ACADEMY 11

QUIZ COMPILATIONS – JULY PART 1

First statement is correct:

Second statement is incorrect:

‘Producer Responsibility Organisation’ means a professional organisation authorised or

financed collectively or individually by producers, which can take the responsibility for

collection and channelisation of e-waste generated from the ‘end-of-life’ of their products

to ensure environmentally sound management of such e-waste;

It will strengthen EPR. With a PRO, producers do not physically take back the product.

Instead, they support the process financially.

Third statement is correct:

95% of e-waste in India is recycled by the informal sector and only 5% is recycled by the

formal sector.

Q14. With respect to climate change, which of the following correctly defines the term

“Negative Emission”?

a. to remove and permanently store CO₂ from the atmosphere.

b. GHG in the atmosphere which create more pollutants when reacted with particles in

the air

c. Carbon sinks (ocean) which are affecting marine species which is leading to increase

in GHG in the atmosphere

d. When emission is more than restoration of GHG.

Solution: a

Explanation:

What are "negative emissions"?

Page 13: QUIZ COMPILATIONS – JULY PART 1

GOALTIDE IAS ACADEMY 12

QUIZ COMPILATIONS – JULY PART 1

To reach zero net emissions (see: What does “zero net emission” mean?) and limit global

warming to 1.5°C, it is necessary to remove and permanently store CO₂ from the

atmosphere. This is called Carbon Dioxide Removal (CDR). As it is the opposite of emissions,

these practices or technologies are often described as achieving "negative emissions" or

"sinks".

There is a direct link between zero net emissions and CDR: The earlier zero net emissions

are achieved; the less CDR is necessary.

There is also a “Coalition for Negative Emissions”, whose goal is not just to decarbonize, but

to decarbonize while ensuring continued economic progress. To do this, negative emissions

will be essential.

Q15. Sea of Marmara connects:

a. Caspian Sea and Adriatic Sea

b. Aegen Sea and Mediterranean Sea

c. Black Sea and Aegen Sea

d. Black Sea and Adriatic Sea

Solution: c

Explanation:

Page 14: QUIZ COMPILATIONS – JULY PART 1

GOALTIDE IAS ACADEMY 13

QUIZ COMPILATIONS – JULY PART 1

Q16. Consider the following statements:

1. The term ‘budget’ is mentioned in Article 112 of Constitution of India.

2. According to the Constitution it is the responsibility of the finance minister to get the

Budget laid in the Parliament.

Which of the above statements are correct?

a. 1 only

b. 2 only

c. Both 1 and 2

d. Neither 1 nor 2

Solution: d

Explanation:

Abhi ye galat hua, toh its not good at all. Please be careful!!!

The term budget is not there in the constitution. In the constitution it is called Annual

Financial Statements. See below image of the article.

The President (not Finance Minister) shall in respect of every financial year cause to be

laid before both the Houses of Parliament a statement of the estimated receipts and

expenditure of the Government of India for that year, in this Part referred to as the annual

financial statement.

Q17. Consider the following statements with reference to the Scheduled Tribes and Other

Traditional Forest Dwellers (Recognition of Forest Rights) Act, 2006:

1. It defines the Community Forest resource Rights and empowers the Gram Sabhas to

govern community forests.

2. Critical Wildlife Habitats have been defined under the above given Act.

Which of the statements given above is/are correct?

a. 1 only

b. 2 only

c. Both 1 and 2

d. Neither 1 nor 2

Solution: c

Page 15: QUIZ COMPILATIONS – JULY PART 1

GOALTIDE IAS ACADEMY 14

QUIZ COMPILATIONS – JULY PART 1

Explanation:

The Scheduled Tribes and Other Traditional Forest Dwellers (Recognition of Forest Rights)

Act, 2006, also called the FRA Act defines the Community Forest resource Rights. So,

statement (1) is correct.

Statement 2 is also correct.

Page 16: QUIZ COMPILATIONS – JULY PART 1

GOALTIDE IAS ACADEMY 15

QUIZ COMPILATIONS – JULY PART 1

Q18. Consider the following lake.

1. Wular Lake

2. Sambhar Lake

3. Sasthamkotta Lake

Which of the above is/are not freshwater lakes?

a. 1 only

b. 2 only

c. 1 and 2 only

d. 1 and 3 only

Solution: b

Explanation:

“Not” asked. Be careful!!!

Page 17: QUIZ COMPILATIONS – JULY PART 1

GOALTIDE IAS ACADEMY 16

QUIZ COMPILATIONS – JULY PART 1

Q19. Consider the following statements:

1. The oath of the Speaker of Lok Sabha is provided in the Third Schedule to the

Constitution.

2. The oath of the President of India is provided in the third Schedule to the

Constitution.

Which of the statements given above is/are not correct?

a. 1 only

b. 2 only

c. Both 1 and 2

d. Neither 1 nor 2

Solution: c

Explanation:

Not Correct, so be careful.

The Lok Sabha Speaker is one of the Members of the Parliament. He/she takes the oath

along with the other Members in the presence of the pro-tem Speaker. No separate oath of

affirmation is administered to him. So, statement 1 is not correct.

Oath of President is not in third schedule.

Q20. Consider the following statements.

1. The Parliament has the power to formulate rules for the convenient transactions of

business of the government of India.

2. Allocation and Transactions of Business rule 1961 is administered by Speaker and

Chairman of Lok Sabha and Rajya Sabha respectively.

Which of the above statements is/are correct?

a. 1 only

b. 2 only

c. Both 1 and 2

d. Neither 1 nor 2

Solution: d

Page 18: QUIZ COMPILATIONS – JULY PART 1

GOALTIDE IAS ACADEMY 17

QUIZ COMPILATIONS – JULY PART 1

Explanation:

According to article 77(3), the President has the power to formulate rules for the

convenient transactions of business of the government of India. Therefore, statement 1 is

correct.

Accordingly, the president has formulated government of India (Allocation of Business) rule

1961 and GOI (Transactions of Business) Rule 1961 as amended from time to time.

The Cabinet Secretariat is responsible for the administration of the Government of India

(Transaction of Business) Rules, 1961 and the Government of India (Allocation of Business)

Rules 1961, facilitating smooth transaction of business in Ministries/Departments of the

Government by ensuring adherence to these rules. Therefore, statement 2 is incorrect.

Q21. Consider the following statements.

1. The credit balance of any deposit account that is maintained with the banks and has

not been operated for ten years or more is categorized under Prior Sector Lending.

2. It is mandatory for banks to display on their websites, the list of unclaimed

deposits/inoperative accounts which are inactive or inoperative.

Which of the above statements is/are correct?

a. 1 only

b. 2 only

c. Both 1 and 2

d. Neither 1 nor 2

Solution: b

Explanation:

News: Reserve Bank of India (RBI) has notified new norms for interest on the amount left unclaimed with bank after a term deposit matures.

The Depositor Education and Awareness Fund Scheme (DEAF Scheme) was established by

the Reserve Bank of India (RBI) in 2014 for the promotion of depositors’ interest and for

any other related purposes deemed necessary by the RBI.

Why this fund was created?

Amount in Non-operative accounts to be transferred to the Fund.

Page 19: QUIZ COMPILATIONS – JULY PART 1

GOALTIDE IAS ACADEMY 18

QUIZ COMPILATIONS – JULY PART 1

First statement is incorrect.

A specific account is maintained by RBI for Depositor Education and Awareness Fund (DEAF).

RBI has directed the following types of amounts to be transferred to this fund.

• The credit balance of any deposit account that is maintained with the banks and has

not been operated for ten years or more.

• Any amount remaining unclaimed for ten years or more.

Second statement is correct.

As per RBI circular dated 02 February 2015, it is mandatory for banks to display on their

websites, the list of unclaimed deposits/inoperative accounts which are inactive or

inoperative for more than 10 years. This activity needs to be carried out every month by

the banks and the unclaimed amounts are to be transferred to RBI by the end of each

month.

Q22. With respect to Mission Innovation Clean Technology Exchange initiative, consider

the following statements.

1. The initiative envisages accelerating clean energy innovations by creating a network of

incubators across member countries.

2. India is launched this initiative.

3. It aims to accelerate action towards the Paris Agreement and net zero pathways.

Select the correct code.

a. 1 only

b. 2 and 3 only

c. 1 and 3 only

d. 1, 2 and 3

Solution: d

Explanation:

Mission Innovation Clean Technology Exchange is a global initiative to catalyze action and

investments in R&D and campaigns to make clean energy affordable, attractive, and

accessible to everyone within the next decade. It hopes to align progress with goals of the

Page 20: QUIZ COMPILATIONS – JULY PART 1

GOALTIDE IAS ACADEMY 19

QUIZ COMPILATIONS – JULY PART 1

Paris Climate Change Agreement and identify scalable pathways to net zero carbon

emissions.

India launched the Mission Innovation CleanTech Exchange, a global initiative, that will

create a network of incubators across member countries to accelerate clean energy

innovation.

The initiative brings together governments, public authorities, corporates, investors, and

academia to achieve these goals, and has become the primary inter-governmental platform

for strengthening global cooperation on clean energy innovation as part of an urgent and

lasting response to climate change.

Q23. 'Sankalp se Siddhi'- Village and Digital Connect drive is related to:

a. Improving livelihood of tribals.

b. establishing a self-sustaining network of CSC centres in Gram Panchayats

c. Digitization of APMCs in villages

d. Digitization of schools in rural areas

Solution: a

Explanation:

TRIFED launched the ‘Sankalp Se Siddhi – Village and Digital Connect’ drive to transform the

country’s tribal ecosystem. Through this programme, TRIFED aims to activate Van Dhan

Vikas Kendras (VDVKs) SHGs located in the tribal villages. It is a 100-day initiative (starting

from April 01, 2021), where 150 teams (10 in each region from TRIFED and state

agencies/mentoring agencies/partners) will visit ten villages each.

Q24. Consider the following pairs.

Places in news feature

1. Bagram Iran

2. Gomti Tributary of Ganges

Which of the above pairs is/are correct?

a. 1 only

b. 2 only

c. Both 1 and 2

d. Neither 1 nor 2

Solution: b

Explanation:

Places in news feature

Bagram Afghanistan

Gomti Tributary of Ganges

Page 21: QUIZ COMPILATIONS – JULY PART 1

GOALTIDE IAS ACADEMY 20

QUIZ COMPILATIONS – JULY PART 1

Q25. As we move away from Tropic of Cancer to Himalayas, which of the following places

will be the southernmost place?

a. Raipur

b. Gandhinagar

c. New Delhi

d. Bhopal

Solution: c

Explanation:

Bhopal, Raipur and Gandhinagar are south to the Tropic of Cancer. So, we are left with

Delhi.

The Tropic of Cancer passes through eight states in India: Gujarat (Jasdan), Rajasthan

(Kalinjarh), Madhya Pradesh (Shajapur), Chhattisgarh (Sonhat), Jharkhand (Lohardaga), West

Bengal (Krishnanagar), Tripura (Udaipur) and Mizoram (Champhai).

Page 22: QUIZ COMPILATIONS – JULY PART 1

GOALTIDE IAS ACADEMY 21

QUIZ COMPILATIONS – JULY PART 1

Q26. Which principle among the following was added to the Directive Principles of State

Policy by the 42nd Amendment to the Constitution?

a. Equal pay for equal work for both men and women

b. Participation of workers in the management of industries

c. Right to work, education and public assistance

d. Securing living wage and human conditions of work to workers

Solution: B

Examination:

2017 UPSC PRELIMS QUESTION.

The 42nd Amendment added new Directive Principles, viz Article 39A, Article 43A and

Article 48A.

Article 43A deals with ‘Participation of workers in management of industries’.

Page 23: QUIZ COMPILATIONS – JULY PART 1

GOALTIDE IAS ACADEMY 22

QUIZ COMPILATIONS – JULY PART 1

Q27. Sculpture of Buddhist goddess Marichi, an example of the process of integration of

different religious and practices, who symbolizes aspects of the sun can be found at:

a. Kushinagar. Uttar Pradesh

b. Bodh Gaya, Bihar

c. Sarnath, Uttar Pradesh

d. Dhauli, Odisha

Solution: b

Explanation:

It can be found in Bodh Gaya, Bihar. Again, from NCERT

Page 24: QUIZ COMPILATIONS – JULY PART 1

GOALTIDE IAS ACADEMY 23

QUIZ COMPILATIONS – JULY PART 1

Q28. As per the provisions mentioned in the Constitution, consider the following

statements.

1. An election to fill a vacancy caused by the expiration of the term of office of Vice-

President shall be completed before the expiration of the term.

2. The person elected to fill the vacancy of Vice President shall be entitled to hold office

for the full term of five years from the date on which he enters upon his office.

Select the correct code.

a. 1 only

b. 2 only

c. Both 1 and 2

d. Neither 1 nor 2

Solution: c

Explanation:

Page 25: QUIZ COMPILATIONS – JULY PART 1

GOALTIDE IAS ACADEMY 24

QUIZ COMPILATIONS – JULY PART 1

Q29. Which of the following is the constitutional authority in India, vested with the power of

declaring certain castes and tribes as the Scheduled Castes (SCs) and Scheduled Tribes (STs)

in a State?

a. The Prime Minister of India

b. The Governor of a State concerned

c. The President of India

d. The Chairman of the National Commission For Scheduled Castes / National

Commission For Scheduled Tribes

Solution: c

Explanation:

The Constitution of India does not specify the castes or tribes which are to be called the

Scheduled Castes or the Scheduled Tribes. It leaves to the President of India, the power to

specify as to which castes or tribes in each State and Union Territory are to be treated as the

SCs and STs.

Q30. Consider the following statements.

1. When a vacancy is created in the office of the Vice President, then the functions of

the Vice President are performed by such person as designated by the President of

the India.

2. The Procedure for the removal of the Vice President cannot be initiated in the Lok

Sabha.

Page 26: QUIZ COMPILATIONS – JULY PART 1

GOALTIDE IAS ACADEMY 25

QUIZ COMPILATIONS – JULY PART 1

Which of the above statements is/are correct?

a. 1 only

b. 2 only

c. Both 1 and 2

d. Neither 1 nor 2

Solution: b

Explanation:

The Constitution is silent on who performs the duties of the Vice-President when a vacancy

occurs in the office of the Vice President of India. See below in the image, what Constitution

talks about vacancy.

It is true that the procedure for the removal of the Vice-President cannot be initiated in the

Lok Sabha. A Vice President may be removed from his office by a resolution of the council

of States passed by a majority of all the then members of the council and agreed to by the

House of the People. Read article 67:

Q31. With respect to Open Network for Digital Commerce (ONDC), consider the following

statements.

1. ONDC project has been started for SEZs to digitize the entire value chain and ease the

cross-border flow of credits.

2. The Project will be implemented by Quality Council of India (QCI).

Select the correct code.

Page 27: QUIZ COMPILATIONS – JULY PART 1

GOALTIDE IAS ACADEMY 26

QUIZ COMPILATIONS – JULY PART 1

a. 1 only

b. 2 only

c. Both 1 and 2

d. Neither 1 nor 2

Solution: b

Explanation:

Department for Promotion of Industry and Internal Trade (DPIIT), Ministry of Commerce

and Industry has initiated a project on Open Network for Digital Commerce (ONDC).

ONDC aims at promoting open networks developed on open-sourced methodology, using

open specifications and open network protocols independent of any specific platform.

ONDC is expected to digitize the entire value chain, standardize operations, promote

inclusion of suppliers, derive efficiencies in logistics and enhance value for consumers.

The task has been assigned to Quality Council of India (QCI).

Link: https://pib.gov.in/PressReleasePage.aspx?PRID=1732949

Q32. Consider the following Pairs.

Places in news Location

1. Sattal Lake Sikkim

2. Buxwaha protected forest region

Madhya Pradesh

Which of the above pairs is/are correct?

a. 1 only

b. 2 only

c. Both 1 and 2

d. Neither 1 nor 2

Solution: b

Explanation:

Places in news Location

Sattal Lake Uttarakhand

Buxwaha protected forest region

Madhya Pradesh

Q33. In the context of Pradhan Mantri Formalization of Micro food processing Enterprises

(PMFME) Scheme, consider the following statements.

1. It aims to enhance the competitiveness of existing individual micro-enterprises in the

unorganized segment of the food processing industry.

Page 28: QUIZ COMPILATIONS – JULY PART 1

GOALTIDE IAS ACADEMY 27

QUIZ COMPILATIONS – JULY PART 1

2. Micro food processing units will be directly assisted with credit linked subsidy.

3. Ministry of MSME will implement the scheme.

Which of the above statements is/are correct?

a. 1 and 2 only

b. 2 and 3 only

c. 1 only

d. 1 and 3 only

Solution: a

Explanation:

Introduction of any scheme is very important:

This scheme is designed to address the challenges faced by the micro enterprises and to tap

the potential of groups and cooperatives in supporting the upgradation and formalization of

these enterprises.

About the scheme: Third statement is incorrect.

Ministry of Food Processing Industry (MoFPI) has launched the Pradhan Mantri

Formalization of Micro food processing Enterprises (PMFME) scheme under the

Aatmanirbhar Bharat Abhiyan with the aim to enhance the competitiveness of existing

individual micro-enterprises in the unorganized segment of the food processing industry

and promote formalization of the sector.

Page 29: QUIZ COMPILATIONS – JULY PART 1

GOALTIDE IAS ACADEMY 28

QUIZ COMPILATIONS – JULY PART 1

The scheme to be implemented over a period of five years from 2020-21 to 2024-25 with a

total outlay of Rupees 10,000 crore. The expenditure under the scheme would to be shared

in 60:40 ratio between Central and State Governments, in 90:10 ratio with North Eastern

and Himalayan States, 60:40 ratio with UTs with legislature and 100% by the Center for

other UTs.

Objectives

The objectives of scheme are to build capability of microenterprises to enable:

• Increased access to credit by existing micro food processing entrepreneurs, FPOs, Self

Help Groups and Co-operatives;

• Integration with organized supply chain by strengthening branding & marketing;

• Support for transition of existing 2,00,000 enterprises into formal framework;

• Increased access to common services like common processing facility, laboratories,

storage, packaging, marketing and incubation services;

• Strengthening of institutions, research and training in the food processing sector; and

• Increased access for the enterprises, to professional and technical support.

Coverage

Under the scheme, 2,00,000 micro food processing units will be directly assisted with credit

linked subsidy. Adequate supportive common infrastructure and institutional architecture

will be supported to accelerate growth of the sector.

Q34. With respect to Energy Compact Goals, consider the following statements.

1. These goals are legally required under the Paris Agreement to address national

climate ambitions and targets.

2. NTPC became first energy company to declare its Energy Compact goals.

Which of the above statements is/are correct?

a. 1 only

b. 2 only

c. Both 1 and 2

d. Neither 1 nor 2

Solution: b

Explanation:

NTPC Limited, India’s largest power generating company under Ministry of Power has

become first energy company in energy domain in India to declare its Energy Compact

goals as part of UN High-level Dialogue on Energy (HLDE).

Energy Compacts:

Why do we need Energy Compacts?

Page 30: QUIZ COMPILATIONS – JULY PART 1

GOALTIDE IAS ACADEMY 29

QUIZ COMPILATIONS – JULY PART 1

Energy is key to the achievement of the 2030 Agenda for Sustainable Development and the

Paris Agreement on Climate Change. However, we are falling behind and current actions on

SDG7 are insufficient, too fragmented, and lacking accountability. There is no platform to

bring together SDG7 related commitments and actions from all stakeholders. By bringing

together all commitments and actions, we can increase accountability and identify gaps

where more commitment is needed and help accelerate action by raising ambitions.

What’s the difference between Energy Compacts and NDCs? First statement is incorrect.

NDCs address Member State’s national climate ambitions and targets that are legally

required under the Paris Agreement, focusing on a country’s emissions profile from the

economy as a whole. On the other hand, recognizing energy’s fundamental role in

emissions, Energy Compacts are intended to reinforce and complement the achievement of

the Paris Agreement, and contribute to Member State’s enhanced NDC and long-term goals.

Q35. Consider the following statements:

1. New Delhi lies in a northern latitude with respect to Kathmandu.

2. Nepal borders with 5 Indian States.

Select the correct code.

a. 1 only

b. 2 only

c. Both 1 and 2

d. Neither 1 nor 2

Solution: c

Explanation:

Statement 1 is correct:

New Delhi lies in 28-degree North latitude, whereas Kathmandu lies in 27- degree north

latitude.

Page 31: QUIZ COMPILATIONS – JULY PART 1

GOALTIDE IAS ACADEMY 30

QUIZ COMPILATIONS – JULY PART 1

Statement 2 is correct:

Nepal borders with Uttarakhand, Uttar Pradesh, Bihar, West Bengal and Sikkim.

Q36. The principal materials used in Lithium -Ion – Batteries are:

1. Cobalt,

2. Manganese

3. Graphite

Select the correct code.

a. 1 and 2 only

b. 3 only

c. 2 only

d. 1, ,2 and 3

Solution: d

Explanation:

Page 32: QUIZ COMPILATIONS – JULY PART 1

GOALTIDE IAS ACADEMY 31

QUIZ COMPILATIONS – JULY PART 1

According to a recent report by the United Nations Conference on Trade and Development

(UNCTAD), the supply of raw materials to produce rechargeable batteries, crucial to move to

cleaner energy, is uncertain.

The report underlined lithium, natural graphite and manganese as critical raw materials for

the manufacture of rechargeable batteries.

Reason for Increasing Demand for Rechargeable Batteries:

a. As electric vehicles (EVs) have gradually been integrated into global transportation,

there has been a rapid growth in demand for rechargeable batteries.

b. This will consequently lead to an increase in the demand for raw material used in

manufacturing these batteries.

c. The demand of raw materials used to manufacture rechargeable batteries will also

grow rapidly as other sources of energy lose their sheen.

d. Alternative sources of energy such as electric batteries will become even more

important as investors grow more wary of the future of the oil industry.

Q37. Which among the following are the points of differences between the Inner Core and

the Outer Core of the Earth?

1. Composition

2. State of Matter

3. Direction of Spin

4. Thickness

5. Temperature

Select the correct answer using the code given below:

a. 1 and 3 only

b. 1, 2 and 4 only

c. 2, 3, 4 and 5 only

d. 1, 2, 3, 4 and 5

Answer: (c)

Explanation:

Page 33: QUIZ COMPILATIONS – JULY PART 1

GOALTIDE IAS ACADEMY 32

QUIZ COMPILATIONS – JULY PART 1

Point (1) is not correct: Both the inner and the outer cores are composed of nickel and iron.

Point (2) is correct: The liquid outer core is composed of a molten mix of iron and nickel,

whereas the inner core is in solid state of matter.

Point (3) is correct: The inner core rotates in an easterly direction, at a speed greater than

the Earth ‘s rotation, whereas the outer core rotates in a westerly direction. This movement

of both the cores is pulled by the Earth ‘s magnetic field.

Point (4) is correct: The thickness of inner core is 1200 km whereas the thickness of outer

core is 2250 km.

Point (5) is correct: The temperature of inner core is 5000° C whereas the temperature of

outer core is 2200° C.

Q38. How does National Biodiversity Authority (NBA) help in protecting the Indian

agriculture?

1. NBA advise the State Governments in the selection of areas of biodiversity

importance to be notified as heritage sites and measures for the management of

such heritage sites.

2. NBA directly monitors and supervises the scientific research on genetic modification

of crop plants.

3. Application for intellectual Property Rights related to genetic / biological resources

cannot be made without the approval of NBA.

Which of the statements given above is/are correct?

a. 1 only

b. 2 and 3 only

c. 1 and 3 only

d. 1, 2 and 3

Solution: c

Explanation:

Page 34: QUIZ COMPILATIONS – JULY PART 1

GOALTIDE IAS ACADEMY 33

QUIZ COMPILATIONS – JULY PART 1

Second is wrong.

The top biotech regulator in India for Genetically Modified Organisms is Genetic

Engineering Appraisal Committee (GEAC).

Q39. In the context of Indian Polity, consider the following statements regarding Tribes

Advisory Council (TAC).

1. Tribes Advisory Council is a Constitutional body under Sixth Schedule.

2. Three-Fourth member of the council shall be the representatives of the Scheduled

Tribes in the Legislative Assembly of the State.

3. The Governor has been empowered to make rules and regulations regarding the

number of members of Tribal Advisory Council and their appointment.

Which of the above statement is/are correct?

a. 2 only

b. 2 and 3 only

c. 3 only

d. 1, 2 and 3

Solution: b

Explanation:

Tribes Advisory Council is a Constitutional body under FIFTH Schedule.

So, what is in sixth schedule?? It is Constitution of district councils and regional councils.

So, remember this difference.

Members are three-fourth. See below now.

Page 35: QUIZ COMPILATIONS – JULY PART 1

GOALTIDE IAS ACADEMY 34

QUIZ COMPILATIONS – JULY PART 1

Q40. Which of the following sites is/are the basins of Shale gas reserves in India?

1. Cauvery basin

2. Godavari basin

3. Vindhyan basins

4. Damodar Valley Basins

Select the correct code.

a. 1 and 2 only

b. 3 and 4 only

c. 1, 2 and 4 only

d. 1, 2, 3 and 4

Solution: d

Explanation:

Shale gas has abundant reserves around the world, which may be sufficient to meet the

demand of clean energy for many years to come. Shale gas is found in unconventional

reservoirs typically trapped in shale rock, having low permeability, originally deposited as

clay and silt.

Page 36: QUIZ COMPILATIONS – JULY PART 1

GOALTIDE IAS ACADEMY 35

QUIZ COMPILATIONS – JULY PART 1

Q41. The Chief Information Commissioner and Information Commissioners shall be

appointed by:

a. Committee consisting of Prime Minister, Leader of Opposition in the Lok Sabha and

Union Cabinet Minister to be nominated by the Prime Minister

b. Committee consisting of Prime Minister, Leader of Opposition of both Houses and

Union Cabinet Minister to be nominated by the Prime Minister

c. Committee consisting of Prime Minister, Leader of Opposition in the Lok Sabha and

Home Minister

d. None of the Above.

Solution: d

Explanation:

Dimag khula rakhna h. Over confident ni hona h. Rest everything is fine.

Q42. Consider the following statements.

1. Southeast Asia contains about half of all tropical mountain forests.

2. Conversion of higher-elevation forest to cropland is accelerating at an

unprecedented rate throughout the region.

Which of the above statements is/are correct?

a. 1 only

b. 2 only

c. Both 1 and 2

d. Neither 1 nor 2

Solution: c

Explanation:

Southeast Asia is home to roughly half of the world’s tropical mountain forests. These

highland ecosystems support massive carbon stores and tremendous biodiversity, including

a host of species that occur nowhere else on the planet.

But new evidence suggests these havens are in grave danger. Conversion of higher-

elevation forest to cropland is accelerating at an unprecedented rate throughout the

region, according to findings published June 28 in Nature Sustainability.

Page 37: QUIZ COMPILATIONS – JULY PART 1

GOALTIDE IAS ACADEMY 36

QUIZ COMPILATIONS – JULY PART 1

Between 2001 and 2019, the researchers calculated that Southeast Asia had lost 610,000

square kilometers (235,500 square miles) of forest — an area larger than Thailand. Of this

loss, 31% occurred in mountainous regions, equivalent to 189,100 km2 (73,000 mi2) of

highland forest converted to cropland and plantation in less than two decades.

Particularly prominent shifts to mountain forest loss were found in north Laos, northeast

Myanmar, and east Sumatra and Kalimantan in Indonesia — the country that experienced

the most overall forest loss.

Q43. With reference to Agroforestry in India, consider the following statements.

1. Agroforestry practices also enhance soil organic carbon

2. Coastal and Humid climatic zones are not suitable for Agroforestry.

3. India has launched National Agroforestry Policy to increase area under Agroforestry.

Which of the above statements is/are correct?

a. 1 and 3 only

b. 2 and 3 only

c. 1 only

d. 1, 2 and 3

Solution: a

Explanation:

First statement is correct.

Agroforestry systems, which integrate trees in agricultural landscapes, have been

recognised by scientists for their role in mitigating climate change by acting as a carbon sink:

trees sequester atmospheric carbon in their biomass. Agroforestry practices can also

enhance soil organic carbon, which is a component of soil organic matter and is the largest

carbon stock in terrestrial ecosystems.

Third statement is correct.

Recognising the potential of agroforestry in meeting various developmental and

environmental goals, India launched a National Agroforestry Policy in 2014. By increasing

the area under agroforestry, the policy aims to address the increasing demand for timber,

food, fuel, fodder, fertiliser and fibre whilst creating employment and generating income.

Second statement is incorrect.

As pe the study conducted in 2019, 60 percent of coastal, 40 percent of humid, 56 percent

of sub-humid, 31 percent of semi-arid and 2.8 percent of the land in arid areas are highly

suitable for agroforestry.

Page 38: QUIZ COMPILATIONS – JULY PART 1

GOALTIDE IAS ACADEMY 37

QUIZ COMPILATIONS – JULY PART 1

Q44. Bryum bharatiensis, a new species discovered first time in 40 years in Antarctica

region by India is:

a. Grasshopper

b. Brown algae

c. moss species

d. Frog species

Solution: c

Explanation:

Page 39: QUIZ COMPILATIONS – JULY PART 1

GOALTIDE IAS ACADEMY 38

QUIZ COMPILATIONS – JULY PART 1

A team of scientists from Punjab has discovered a new moss species in eastern Antarctica

and named it Bryum bharatiensis as a tribute to goddess Saraswati, also known as Bharati.

One of India’s Antarctic stations is also called Bharati.

The Indian Antarctic Mission was begun in 1981.This is the first and the only plant species

that the mission has discovered in 40 years.

Q45. Consider the following.

1. Bab-el-Mandeb

2. Gulf of Aqaba

3. Riyadh

Arrange the above from North to South.

a. 2-1-3

b. 1-2-3

c. 2-3-1

d. 1-3-2

Solution: c

Explanation:

Q46. Consider the following provisions with respect to India.

1. Ministers shall be appointed by the President on the advice of the Prime Minister.

Page 40: QUIZ COMPILATIONS – JULY PART 1

GOALTIDE IAS ACADEMY 39

QUIZ COMPILATIONS – JULY PART 1

2. Appointment of ad hoc judges If at any time there should not be a quorum of the

Judges of the Supreme Court available to hold or continue any session of the Court

3. Uniform Civil Code

Which of the above provisions are mentioned in the Constitution?

a. 1 and 2 only

b. 2 and 3 only

c. 1 and 3 only

d. 1, 2 and 3

Solution: d

Explanation:

All the above provisions are mentioned in the Constitution. We are pasting all the three

articles. You will read all the provisions of the article given below.

Page 41: QUIZ COMPILATIONS – JULY PART 1

GOALTIDE IAS ACADEMY 40

QUIZ COMPILATIONS – JULY PART 1

Q47. With reference to the medieval history of India, the terms Dahsala, Batai and Nasaq

are related to:

a. Administrative posts during rule of Mughal dynasty.

b. Land revenue system during Akbar reign

c. Merchant guilds during Guptas reign

d. Irrigation taxes during Harshvardhan reign

Solution: b

Explanation:

All three terms are related to land revenue system during the Akbar Reign.

In 1580, Akbar instituted a new system called Dahsala. Under this the average produce of

different crops and their average prices prevailing over the last 10 years were calculated and

l/3rd the average produce fixed in rupees per bigha was demanded as the state’s share.

Galla bakhshi means crop-division and in Hindi it is known as batai or bhaoli. It is, according

to Abu-1-Fazl, a mode of assessment by which the state’s demand used to be assessed on

the basis of the division of crop by agreement in presence of the parties. According to the

Galla-Bakshi system the produce of the farmers was divided between the Government and

Page 42: QUIZ COMPILATIONS – JULY PART 1

GOALTIDE IAS ACADEMY 41

QUIZ COMPILATIONS – JULY PART 1

the farmers in the ratio settled between them. After cutting the crops, they were bound in

bundles and then these bundles were divided between the farmers and the Government.

Third system used under Akbar’s reign was nasaq. It means a rough calculation of the

amount of the payable by the peasant on the basis of what he has been paying in the past.

Q48. If there is rise in unemployment due to the adoption of Artificial Intelligence and

Industrial Revolution 4.0, it will be best described as:

a. Seasonal Unemployment

b. Structural Unemployment

c. Frictional Unemployment

d. Cyclical Unemployment

Solution: b

Explanation:

Structural unemployment results from mismatches between the skills required for

available jobs and the skills held by the unemployed. Even when there is plenty of job

availability, this mismatch means the unemployed cannot access jobs that fit their skill sets.

It is caused by fundamental shift in an economy and due to major changes in technology,

competition and government policy.

Q49. The power to establish a common court for two or more states is vested in:

a. Parliament

b. President of India

c. Chief Justice of India

d. Supreme Court of India

Solution: a

Explanation:

The power to establish a common court for two or more states is vested in Parliament of

India.

Q50. Consider the following statements.

Page 43: QUIZ COMPILATIONS – JULY PART 1

GOALTIDE IAS ACADEMY 42

QUIZ COMPILATIONS – JULY PART 1

1. Thanjavur painting are associated with the Nayaka Dynasty.

2. Thanjavur painting has been recognized as a Geographical indication rights by the

Government of India.

Which of the above statements is/are correct?

a. 1 only

b. 2 only

c. Both 1 and 2

d. Neither 1 nor 2

Solution: c

Explanation:

Tanjore Paintings date back to 17th century, when Tanjore was under the Nayaka Dynasty.

They represent a form of Classical South Indian paintings that was practiced from several

centuries in the town of Thanjavur in Tamil Nadu.

Thanjavur Paintings have been recognized with Geographical Indication rights by the

Government of India in 2007.

Q51. Consider the following pairs.

COVID Variants of Concern (VOC) Earliest documented samples

1. Alpha United Kingdom

2. Gamma South Africa

3. Delta India

Which of the above pairs is/are correct?

a. 1 and 3 only

b. 2 and 3 only

c. 1 and 2 only

d. 3 only

Solution: a

Explanation:

The WHO has classified four variants as VoC viz; Alpha (B.1.1.7), Beta (B.1.351), Gamma

(P.1), and Delta (B.1.617.2).

Page 44: QUIZ COMPILATIONS – JULY PART 1

GOALTIDE IAS ACADEMY 43

QUIZ COMPILATIONS – JULY PART 1

Q52. Consider the following statements.

1. India has adopted National Chemical Policy 2019 for regulation and effective

governance of chemicals.

2. India has legally binding controls on the production, import, sale and use of lead

paints.

Which of the above statements is/are correct?

a. 1 only

b. 2 only

c. Both 1 and 2

d. Neither 1 nor 2

Solution: b

Explanation:

Second statement is correct:

According to the report by the United Nations Environment Programme (UNEP), only 13

countries have laws which prescribe that lead concentration should not be more than 90

Page 45: QUIZ COMPILATIONS – JULY PART 1

GOALTIDE IAS ACADEMY 44

QUIZ COMPILATIONS – JULY PART 1

particles per million (ppm). They are: India, the United States (US), Bangladesh, Canada,

Cameroon, China, Ethiopia, Iraq, Israel, Jordan, Kenya, Nepal and the Philippines.

First statement is correct:

Country’s national chemical policy has been pending since 2012. There is a need for a

comprehensive law in the country to regulate chemical use, production and safety.

Q53. Consider the following statements.

1. In India, the real estate sector is the second-highest employment generator after the

agriculture sector.

2. The real state sector contribution to GDP is more than 3 percent.

3. To boost urbanization and investments, the government has allowed FDIs of up to

100% for all townships & settlement development projects.

Which of the above statements is/are correct?

a. 1 and 2 only

b. 2 and 3 only

c. 1 and 3 only

d. 1, 2 and 3

Solution: d

Explanation:

In India, the real estate sector is the second-highest employment generator after the

agriculture sector. In 2021, the sector was responsible for employing >5.5 crore people,

garnering a total of 11% of all employment opportunities in the country.

The sector is also a substantial contributor to the national gross domestic product (GDP). As

of 2021, the sector’s contribution to the total GDP was valued at Rs. 1,483,745 crore (US

$200 billion), which was equivalent to a 7% contribution share in the country’s overall

GDP.

To boost urbanization and investments, the government has allowed FDIs of up to 100%

for all townships & settlement development projects and relaxed the minimum

capitalization requirements for FDI investments from Rs. 7.4 crore (US $10 million) to Rs. 3.7

crore (US $5 million). Both these measures are expected to stimulate FDI investments over

the next five years, with India expected to attract capital infusion of ~Rs. 59,350 crore (US

$8 billion) by FY25.

Q54. With reference to 'Bamboo Oasis on Lands in Drought' (BOLD) project, consider the

following statements.

1. It aims to reduce land degradation and preventing desertification in the country.

2. TRIFED will implement this Project.

Page 46: QUIZ COMPILATIONS – JULY PART 1

GOALTIDE IAS ACADEMY 45

QUIZ COMPILATIONS – JULY PART 1

Which of the above statements is/are correct?

a. 1 only

b. 2 only

c. Both 1 and 2

d. Neither 1 nor 2

Solution: a

Explanation:

The Khadi and Village Industries Commission launched a project named 'Bamboo Oasis on

Lands in Drought' to support the local bamboo-based industries in rural India. Second

statement is correct.

Project BOLD, which seeks to create bamboo-based green patches in arid and semi-arid land

zones, is aligned with Prime Minister Shri Narendra Modi’s call for reducing land

degradation and preventing desertification in the country. First statement is correct.

India is a signatory to the UN Convention to Combat Desertification (UNCCD). In his keynote address at the UN High-Level Dialogue on Desertification, Land Degradation and Drought on June 14, Prime Minister Modi gave a clarion call of restoring 26 million hectares of degraded land by 2030. This assumes great significance as nearly 30 percent of land in India is undergoing desertification at a rapid pace.

Under the project, 5,000 saplings of special bamboo species Bambusa Tulda and Bambusa

Polymorpha brought from Assam have been planted over 25 bigha (16 acres) of vacant arid

Gram Panchayat land.

Q55. Consider the following seas.

1. Aegean Sea

2. Black Sea

3. Adriatic Sea

Arrange the above seas from West to East.

a. 2-3-1

b. 3-1-2

c. 3-2-1

d. 2-1-3

Solution: B

Explanation:

Adriatic Sea was already asked in Prelims 2019.

Correct order from West to East is:

Adriatic- Aegean- Black

Page 47: QUIZ COMPILATIONS – JULY PART 1

GOALTIDE IAS ACADEMY 46

QUIZ COMPILATIONS – JULY PART 1

See the map now.

Q56. Who among the following has the power to request a retired Supreme Court Judge to

act as a Judge of the Supreme Court for a temporary period?

a. President of India with the previous consent of the Chief Justice of India.

b. Chief Justice of India with the previous consent of the President.

c. It is the power of the collegium to suggest a name to the President.

d. There is no such provision in the Constitution of India

Solution: b

Explanation:

As per provisions of the Article 128, Chief Justice of India, with the previous consent of the

President, request a retired Judge of the Supreme Court High Court, who is duly qualified for

appointment as a Judge of the Supreme Court, to sit and act as a Judge of the Supreme

Court.

Q57. Which of the following reasons explain the importance of Foreign Direct Investment

(FDI) for India?

1. For overhauling its infrastructure growth

Page 48: QUIZ COMPILATIONS – JULY PART 1

GOALTIDE IAS ACADEMY 47

QUIZ COMPILATIONS – JULY PART 1

2. For maintaining the value of Rupee

3. Creating jobs

4. Breaking domestic monopolies

Select the correct answer using the code given below:

a. 1 and 2 only

b. 1, 3 and 4 only

c. 2 and 3 only

d. 1, 2, 3 and 4

Solution: d

Explanation:

Foreign Direct Investment (FDI) is when a company takes controlling ownership in a business

entity in another country. With FDI, foreign companies are directly involved with day-to-day

operations in the other country. This means they aren’t just bringing money with them, but

also knowledge, skills and technology. Generally, FDI takes place when an investor

establishes foreign business operations or acquires foreign business assets, including

establishing ownership or controlling interest in a foreign company.

FDI is important as India would require huge investments in the coming years to overhaul its

infrastructure sector to boost growth. So, statement (1) is correct.

Healthy growth in foreign inflows helps maintain the balance of payments and the value of

the Rupee. The Current Account Deficit in Balance of Payments is met by India’s Capital

Account surplus. So, statement (2) is correct.

Creation of jobs is the most obvious advantage of FDI. It is also one of the most important

reasons why a nation, especially a developing one, looks to attract FDI. Increased FDI boosts

manufacturing as well as the services sector. This, in turn, creates jobs and helps reduce

unemployment among the educated youth - as well as skilled and unskilled labour - in the

country. Increased employment translates to increased incomes and equips the population

with enhanced buying power. This boosts the economy of the country. So, statement (3) is

correct.

By facilitating the entry of foreign organizations into the domestic marketplace, FDI helps

create a competitive environment, as well as break domestic monopolies. A healthy

competitive environment pushes firms to enhance their processes and product offerings

continuously, thereby fostering innovation. Consumers also gain access to a wider range of

competitively priced products. So, statement (4) is correct.

Q58. Consider the following options.

1. Member of State Legislature

2. Candidate for Election to Parliament (not elected, just nominated candidate)

Page 49: QUIZ COMPILATIONS – JULY PART 1

GOALTIDE IAS ACADEMY 48

QUIZ COMPILATIONS – JULY PART 1

3. Judges of High Court

Who’s of the above Oaths are included in the third schedule of Indian constitution?

a. 1 only

b. 1 and 3 only

c. 2 and 3 only

d. 1, 2 and 3

Solution: d

Explanation:

Many of you might have confusion in option b, whether candidate for the election to

Parliament is part of third schedule or not. Yes, nominated candidate for the both-elections

to Parliament and State Legislature, are part of third schedule.

Their oath and affirmations of the above three are mentioned below:

a. Form of oath or affirmation to be made by a member of the Legislature of a State:

“I, A.B., having been elected (or nominated) a member of the Legislative Assembly (or

Legislative Council), do swear in the name of God that I will bear true faith and allegiance to

the Constitution of India as by law established, that I will uphold the sovereignty and

integrity of India and that I will faithfully discharge the duty upon which I am about to

enter.”

b. Form of oath or affirmation to be made by a candidate for election to Parliament:

“I, A.B., having been nominated as a candidate to fill a seat in the Council of States (or the

House of the People) do swear in the name of God that I will bear true faith and allegiance

to the Constitution of India as by law established and that I will uphold the sovereignty and

integrity of India.”

c. Form of oath or affirmation to be made by the Judges of a High Court:

“I, A.B., having been appointed Chief Justice (or a Judge) of the High Court do swear in the

name of God that I will bear true faith and allegiance to the Constitution of India as by law

established, [that I will uphold the sovereignty and integrity of India, inserted by 16th

Constitutional amendment, 1963] that I will duly and faithfully and to the best of my ability,

knowledge and judgment perform the duties of my office without fear or favour, affection

or ill-will and that I will uphold the Constitution and the laws.

Q59. Which of the following states is/are part of The Kaladan multimodal project?

a. Manipur only

b. West Bengal and Mizoram

c. West Bengal and Manipur

d. Mizoram only

Page 50: QUIZ COMPILATIONS – JULY PART 1

GOALTIDE IAS ACADEMY 49

QUIZ COMPILATIONS – JULY PART 1

Solution: b

Explanation:

Q60. The term Political Justice is mentioned in Indian Constitution under:

a. Preamble and Fundamental Rights only

b. Preamble and Directive Principles of State Policy only

c. Preamble only

d. Fundamental Rights and Directive Principles of State Policy

Solution: b

Explanation:

Article 38 of the Indian Constitution reads as: The State shall strive to promote the welfare

of the people by securing and protecting as effectively as it may a social order in which

justice, social, economic and political, shall inform all the institutions of the national life.

Preamble also talks about Political Justice.

Page 51: QUIZ COMPILATIONS – JULY PART 1

GOALTIDE IAS ACADEMY 50

QUIZ COMPILATIONS – JULY PART 1

Q61. Carbon Border Adjustment Mechanism (“CBAM”) regulation is proposed by:

a. World Trade Organization

b. European Commission

c. ASEAN

d. China for CPEC project

Solution: b

Explanation:

The European Union (EU) is in the process of establishing a carbon border adjustment

mechanism (CBAM), in the context of the European Green Deal. The CBAM would place a

carbon price on imports of certain goods from outside the EU, as a way to reduce the risk of

“carbon leakage”.

Q62. With respect to India, consider the following statements regarding Digital

Communications Commission.

Page 52: QUIZ COMPILATIONS – JULY PART 1

GOALTIDE IAS ACADEMY 51

QUIZ COMPILATIONS – JULY PART 1

1. The Commission is responsible for Preparing the budget for the Department of

Telecommunications for each financial year.

2. Union minister is the Chairperson of Commission.

3. It has both administrative and financial power of Government of India for carrying out

the work of Department of Communication.

Which of the above statements is/are correct?

a. 1 and 2 only

b. 1 and 3 only

c. 3 only

d. 2 and 3 only

Solution: b

Explanation:

The Telecom Commission was set up by the Government of India dated 11th April, 1989

with administrative and financial powers of the Government of India to deal with various

aspects of Telecommunications.

The Government in 2018 has re-designated the 'Telecom Commission' as the 'Digital

Communications Commission'.

First and third statement is correct.

Page 53: QUIZ COMPILATIONS – JULY PART 1

GOALTIDE IAS ACADEMY 52

QUIZ COMPILATIONS – JULY PART 1

Second statement is correct:

The Secretary to the Government of India in the Department of Telecommunications is the

ex-officio Chairman of the Digital Communications Commission.

Q63. Consider the following statements regarding Miyawaki method in Agriculture.

1. One of the important criteria for this method is, not to use of indigenous species for

plantations.

2. Under the approach, dozens of species are planted in the same area, close to each

other, which ensures that the plants receive sunlight only from the top, and grow

upwards than sideways.

3. The method has been developed by Japanese.

Which of the above statements is/are correct?

a. 1 and 2 only

b. 3 only

c. 2 and 3 only

d. 1, 2 and 3

Solution: c

Explanation:

The Miyawaki method, also called the Potted Seedling Method, is an afforestation

technique that uses native species to create dense, multi-layered forests.

A central tenet in creating these forests is its use of indigenous species for plantations.

First statement is incorrect.

The overall density of the forest is beneficial in lowering temperature, making soil

nutritious, supporting local wildlife and sequestration of carbon.

Under the approach, dozens of native species are planted in the same area, close to each

other, which ensures that the plants receive sunlight only from the top, and grow upwards

than sideways. Second statement is correct.

Invented by and named after Japanese botanist Akira Miyawaki, the ‘Miyawaki Method’ is

a unique technique to grow forests. Third statement is correct.

Q64. Consider the following statements.

1. The Ministries/Departments of the Government are created by the President on the

advice of the Prime Minister under these Rules.

2. Each of the Ministries is assigned to a Minister by the President on the advice of the

Prime Minister.

Which of the above statements is/are correct?

a. 1 only

b. 2 only

Page 54: QUIZ COMPILATIONS – JULY PART 1

GOALTIDE IAS ACADEMY 53

QUIZ COMPILATIONS – JULY PART 1

c. Both 1 and 2

d. Neither 1 nor 2

Solution: c

Explanation:

The Government of India (Allocation of Business) Rules, 1961 are made by the President of

India under Article 77 of the Constitution for the allocation of business of the Government

of India.

The Ministries/Departments of the Government are created by the President on the

advice of the Prime Minister under these Rules.

The business of the Government is transacted in the Ministries/Departments, Secretariats

and offices (referred to as 'Department') as per the distribution of subjects specified in these

Rules.

Each of the Ministries is assigned to a Minister by the President on the advice of the Prime

Minister. Each department is generally under the charge of a secretary to assist the Minister

on policy matters and general administration.

Q65. Consider the following Indian states.

1. Meghalaya

2. Mizoram

3. Manipur

4. Arunachal Pradesh

Which of the above Indian states share boundaries with only two countries?

a. 3 and 4 only

b. 1 and 2 only

c. 2 and 4 only

d. 2 only

Solution: d

Explanation:

Arunachal Pradesh shares boundary with- Bhutan, China and Myanmar

Meghalaya shares boundary only with Bangladesh.

Manipur shares boundary with only Myanmar.

Mizoram shares boundary with Bangladesh and Myanmar. This is only correct option.

Page 55: QUIZ COMPILATIONS – JULY PART 1

GOALTIDE IAS ACADEMY 54

QUIZ COMPILATIONS – JULY PART 1

Q66. The Aitchinson Commission, 1887 is related to:

a. Civil Services reforms

b. Police Reforms

c. Judicial Reforms

d. Army Reforms

Solution: a

Explanation:

In short answer is Civil Servis reform. If you want to read full history of Civil Services in

detail, read below.

We will see History of Commission. You will enjoy reading and very important for your

Prelims Exam and Mains Exam.

a. Civil Servants for the East India Company used to be nominated by the Directors of the

Company and thereafter trained at Haileybury College in London and then sent to

India. Following Lord Macaulay’s Report of the Select Committee of British Parliament,

the concept of a merit based modern Civil Service in India was introduced in 1854.

b. The Report recommended that patronage-based system of East India Company should

be replaced by a permanent Civil Service based on a merit-based system with entry

through competitive examinations.

c. For this purpose, a Civil Service Commission was setup in 1854 in London and

competitive examinations were started in 1855.

Page 56: QUIZ COMPILATIONS – JULY PART 1

GOALTIDE IAS ACADEMY 55

QUIZ COMPILATIONS – JULY PART 1

d. Initially, the examinations for Indian Civil Service were conducted only in London.

Maximum age was 23 years and minimum age was 18 years. The syllabus was

designed such that European Classics had a predominant share of marks.

e. All this made it difficult for Indian candidates. Nevertheless, in 1864, the first Indian,

Shri Satyendra Nath Tagore brother of Shri Rabindranath Tagore succeeded. Three

years later 4 other Indians succeeded.

f. Throughout the next 50 years, Indians petitioned for simultaneous examinations to be

held in India without success because the British Government did not want many

Indians to succeed and enter the ICS.

g. It was only after the First World War and the Montagu Chelmsford reforms that this

was agreed to. From 1922 onwards the Indian Civil Service Examination began to be

held in India also, first in Allahabad and later in Delhi with the setting up of the Federal

Public Service Commission. The Examination in London continued to be conducted by

the Civil Service Commission.

h. Regarding Central Civil Services, the Civil Services in British India were classified as

covenanted and uncovenanted services on the basis of the nature of work, pay-scales

and appointing authority. In 1887, the Aitchinson Commission recommended the

reorganization of the services on a new pattern and divided the services into three

groups-Imperial, Provincial and Subordinate. The recruiting and controlling authority

of Imperial services was the ‘Secretary of State’.

i. With the passing of the Indian Act 1919, the Imperial Services headed by the

Secretary of State for India, were split into two-All India Services and Central

Services. The central services were concerned with matters under the direct control of

the Central Government.

j. The origin of the Public Service Commission in India is found in the First Dispatch of

the Government of India on the Indian Constitutional Reforms on the 5th March,

1919 which referred to the need for setting up some permanent office charged with

the regulation of service matters.

k. Section 96(C) of the Act provided for the establishment in India of a Public Service

Commission which should “discharge, in regard to recruitment and control of the Public

Services in India, such functions as may be assigned thereto by rules made by the

Secretary of State in Council”. No decision was taken on setting up of the body after

passing of 1919 Act.

l. The subject was then referred to the Royal Commission on the Superior Civil Services

in India (also known as Lee Commission) which recommended that the statutory

Public Service Commission contemplated by the Government of India Act, 1919 should

be established without delay.

m. Subsequent to the provisions of Section 96(C) of the Government of India Act, 1919

and the strong recommendations made by the Lee Commission in 1924 for the early

establishment of a Public Service Commission, it was on October 1, 1926 that the

Public Service Commission was set up in India for the first time.

n. Further, the Government of India Act, 1935 envisaged a Public Service Commission for

the Federation and a Provincial Public Service Commission for each Province or group

of Provinces. Therefore, in terms of the provisions of the Government of India Act,

Page 57: QUIZ COMPILATIONS – JULY PART 1

GOALTIDE IAS ACADEMY 56

QUIZ COMPILATIONS – JULY PART 1

1935 and with its coming into effect on 1st April 1937, the Public Service Commission

became the Federal Public Service Commission.

o. With the inauguration of the Constitution of India in January 26, 1950, the Federal

Public Service Commission came to be known as the Union Public Service

Commission, and the Chairman and Members of the Federal Public Service

Commission became Chairman and Members of the Union Public Service Commission

by virtue of Clause (1) of Article 378 of the Constitution.

Q67. Consider the following statements regarding National Security Act, 1980.

1. The National Security Act was promulgated during the Indira Gandhi government.

2. Both Central and State Government has power under this Act to detain a person.

Select the correct code.

a. 1 only

b. 2 only

c. Both 1 and 2

d. Neither 1 nor 2

Solution: c

Explanation:

Page 58: QUIZ COMPILATIONS – JULY PART 1

GOALTIDE IAS ACADEMY 57

QUIZ COMPILATIONS – JULY PART 1

Q68. In which of the following regions ‘Coral triangle’ lies?

a. Indian Ocean

b. Caribbean Sea

c. Equatorial West Pacific Ocean

d. Atlantic Ocean

Solution: c

Explanation:

Q69. Which of the following are envisaged by the Right against Exploitation in the

Constitution of India?

Page 59: QUIZ COMPILATIONS – JULY PART 1

GOALTIDE IAS ACADEMY 58

QUIZ COMPILATIONS – JULY PART 1

1. Prohibition of traffic in human beings and forced labour

2. Abolition of untouchability

3. Protection of the interests of minorities

4. Prohibition of employment of children in factories and mines

Select the correct answer using the code given below:

a. 1, 2 and 4 only

b. 2, 3 and 4 only

c. 1 and 4 only

d. 1, 2, 3 and 4

Solution: c

Explanation:

Abolition of untouchability comes under Right to Equality. Easily eliminate. So, left with

only option c.

Protection of the interests of minorities comes under Cultural and Educational Rights.

Q70. As per the provisions mentioned in the Constitution, consider the following

statements.

1. An election to fill a vacancy caused by the expiration of the term of office of

President shall be completed before the expiration of the term.

2. The person elected to fill the vacancy of the President shall be entitled to hold office

for the full term of five years from the date on which he enters upon his office.

Select the correct code.

a. 1 only

b. 2 only

c. Both 1 and 2

d. Neither 1 nor 2

Solution: c

Explanation:

Page 60: QUIZ COMPILATIONS – JULY PART 1

GOALTIDE IAS ACADEMY 59

QUIZ COMPILATIONS – JULY PART 1

Q71. Consider the following statements regarding Mahatma Gandhi National Rural

Employment Guarantee Act, 2005.

1. Under the Act, priority be given to women in such a way that at least one-third of the

beneficiaries be women.

2. Currently, wage rates are notified based on Consumer Price Index-Agricultural

Labourers (CPI-AL) by the State Government.

3. Employment demand under the MGNREGA decreased as a result of the pandemic

induced lockdown in 2020-21.

Which of the above statements is/are correct?

a. 1 and 2 only

b. 2 and 3 only

c. 1 only

d. 1 and 3 only

Solution: c

Explanation:

First statement is correct.

Second statement is incorrect:

A brief review of the history of how wages are determined is important.

The employment guarantee act gives the central government two options for determining

the MNREGA wage rate. The first is that MNREGA workers are paid the state minimum

wage for agricultural labourers. The second is that the central government notifies separate

wage rates for MNREGA. Till 2008, MNREGA wages were fixed as per the first option. From 1

January 2009, the central government switched to the second option.

Page 61: QUIZ COMPILATIONS – JULY PART 1

GOALTIDE IAS ACADEMY 60

QUIZ COMPILATIONS – JULY PART 1

Now, wage rates for workers under the Mahatma Gandhi National Rural Employment

Guarantee Act (MGNREGA), 2005 are notified and revised annually based on Consumer

Price Index-Agricultural Labourers (CPI-AL) by the Central Government in accordance with

the provisions of Section 6(1) of the Mahatma Gandhi NREGA.

Third statement is incorrect:

Employment demand increased.

Q72. Catalytic Green Finance Facility (CGF) is an initiative of:

a. ASEAN

b. World Economic Forum

c. UNEP

d. Asian Development Bank

Solution: a

Explanation:

Page 62: QUIZ COMPILATIONS – JULY PART 1

GOALTIDE IAS ACADEMY 61

QUIZ COMPILATIONS – JULY PART 1

The Association of Southeast Asian Nations (ASEAN) Catalytic Green Finance Facility

(ACGF) was launched in April 2019 to promote green infrastructure investments in

Southeast Asia.

The ACGF is the only green finance facility focused on developing and increasing climate-

positive projects that is owned by the governments of a regional bloc, in this case the 10

ASEAN member countries.

Q73. Consider the following statements.

1. One of the important reasons for rising lightning events is global warming.

2. The seasonality and severity of lightning may vary for different areas.

3. Lightning events in India has increase in 2020-21 as compared to 2019-2020.

Which of the above statements is/are correct?

a. 1 and 2 only

b. 2 and 3 only

c. 1 and 3 only

d. 1, 2 and 3

Solution: d

Explanation:

First statement is correct.

Climate change may be sparking more lightning across the world, and there is an

increasing scientific evidence pointing to the trend, a study by the Atmospheric Electricity

Group and published in Geophysical Research Letters in 2020.

As per the study, the rise in incidences and intensity of lightning strikes could be due to

global warming and the growth of major urban centers.

Second statement is correct.

Within each state, the seasonality and severity may vary for different areas viz coastal,

hilly, river basin, urban and industrial areas.

Lightning in India: Third statement is correct.

As many as 18.5 million lightning strikes were recorded in India between April 1, 2020 and

March 31, 2021, according to India’s second annual report on lightning released by Lightning

Resilient India Campaign (LRIC) recently.

This is an increase of 34 per cent compared to previous year; at least 13.8 million strikes

were recorded between April 1, 2019 and March 31, 2020.

Q74. “Net-Zero Asset Owner Alliance” is an initiative of:

a. UNEP

b. World Bank

Page 63: QUIZ COMPILATIONS – JULY PART 1

GOALTIDE IAS ACADEMY 62

QUIZ COMPILATIONS – JULY PART 1

c. UNDP Secretariat.

d. World Economic Forum

Solution: a

Explanation:

The members of the Alliance commit to transitioning their investment portfolios to net-zero

GHG emissions by 2050 consistent with a maximum temperature rise of 1.5°C above pre-

industrial temperatures, taking into account the best available scientific knowledge.

Q75. Consider the following countries.

1. Eritrea

2. Yemen

3. Djibouti

4. Somalia

Which of the above countries share/s boundary with Red Sea?

a. 2 only

b. 3 and 4 only

c. 1, 2 and 3 only

d. 1 and 2 only

Solution: c

Explanation:

There are six countries (Saudi Arabia, Yemen, Egypt, Sudan, Eritrea, and Djibouti) bordering

the Red Sea.

Page 64: QUIZ COMPILATIONS – JULY PART 1

GOALTIDE IAS ACADEMY 63

QUIZ COMPILATIONS – JULY PART 1

Q76. Consider the following bodies.

1. Co-operative societies

2. Election Commissioners

3. Solicitor General

Which of the above is/are Constitutional bodies?

a. 1 only

b. 2 only

c. 3 only

d. 1 and 2 only

Solution: b

Explanation:

Right to form Co-operative societies is Constitutional Right. Co-operative societies are not

Constitutional bodies.

Page 65: QUIZ COMPILATIONS – JULY PART 1

GOALTIDE IAS ACADEMY 64

QUIZ COMPILATIONS – JULY PART 1

Election Commissioner is also Constitutional.

Solicitor General is not a Constitutional Body.

Q77. Consider the following statements regarding composition of GST Council.

1. It is a Constitutional Body.

2. The Chairperson of Central Board of Indirect Excise and Customs (CBEC) is the ex-

officio member of GST Council.

3. Secretary, Department of Expenditure, is the ex-officio secretary to the GST Council.

Which of the above statements is/are correct?

a. 1 and 2 only

b. 1 and 3 only

c. 1 only

d. 2 and 3 only

Solution: c

Explanation:

GST Council is a constitutional body for making recommendations to the Union and State

Government on issues related to Goods and Service Tax. Union Finance Minister is the

chairman of GST Council.

As per Article 279A (1) of the amended Constitution, the GST Council has to be constituted

by the President within 60 days of the commencement of Article 279A.

The Chairperson of Central Board of Excise and Customs (CBEC) is a permanent invitee with

no voting rights. He is not a member of Council.

Secretary, Department of Revenue, is the ex-officio secretary to the GST Council.

Page 66: QUIZ COMPILATIONS – JULY PART 1

GOALTIDE IAS ACADEMY 65

QUIZ COMPILATIONS – JULY PART 1

Q78. Consider the following statements about a classical dance form.

1. Tribhanga is a special feature of this dance.

2. The torso movement is very important and is a unique feature of this dance.

3. Maharis and Gotipuas are associated with this dance.

Identify the classical dance.

a. Kuchipudi Dance

b. Odissi dance

c. Sattriya Dance

d. Kathakali dance

Solution: b

Explanation:

The dance is Odissi.

Odissi closely follows the tenets laid down by the Natya Shastra. Facial expressions, hand

gestures and body movements are used to suggest a certain feeling, an emotion or one of

the nine rasas. The techniques of movement are built around the two basic postures of the

Chowk and the Tribhanga.

The torso movement is very important and is a unique feature of the Odissi style. With the

lower half of the body remaining static, the torso moves from one side to the other along

the axis passing through the centre of the upper half of the body.

For centuries maharis were the chief repositories of this dance. The maharis, who were

originally temple dancers came to be employed in royal courts which resulted in the

degeneration of the art form.

Page 67: QUIZ COMPILATIONS – JULY PART 1

GOALTIDE IAS ACADEMY 66

QUIZ COMPILATIONS – JULY PART 1

Around this time, a class of boys called gotipuas were trained in the art, they danced in the

temples and also for general entertainment. Many of today's gurus of this style belong to

the gotipua tradition.

An Odissi orchestra essentially consists of a pakhawaj player (usually the Guru himself), a

singer, a flutist, a sitar or violin player and a manjira player.

Link: http://ccrtindia.gov.in/odissi.php

Q79. ‘Chihil sutun’ or ‘forty-pillared halls’ was a piece of architecture during which Mughal

Ruler?

a. Akbar

b. Shah Jahan

c. Jahangir

d. Aurangzeb

Solution: b

Explanation:

It belongs to Shah Jahan’s rule. We have taken this question from Class 7th, Our Pasts-

Chapter 5 (Rulers and Buildings).

Q80. Consider the following

1. X-Rays

2. WIFI devices

3. Microwaves and Radio waves

Which of the above is/are sources of ionizing radiations?

a. 1 and 2 only

b. 3 only

c. 1 only

d. 1 and 3 only

Solution: c

Explanation:

There are two types of radiations:

a. Ionizing radiations

b. Non-ionizing radiations

These radiations have high penetration power and breaks into macro molecules in the body.

Examples: X-Rays, Gamma rays, Atomic Plants, etc.

Page 68: QUIZ COMPILATIONS – JULY PART 1

GOALTIDE IAS ACADEMY 67

QUIZ COMPILATIONS – JULY PART 1

Non-ionizing radiation is the term given to radiation in the part of the electromagnetic

spectrum where there is insufficient energy to cause ionization.

It includes electric and magnetic fields, radio waves, microwaves, infrared, ultraviolet, and

visible radiation. WIFI devices, Telecom Towers will come under this.

Remember this also:

Longer wavelength, lower frequency waves (heat and radio) have less energy than shorter

wavelength, higher frequency waves (X and gamma rays).

Q81. Consider the following statements.

1. Liquid oxygen has a pale blue colour and is a cryogenic liquid.

2. Liquid oxygen is used as an oxidant for liquid fuels in the propellant systems of missiles

and rockets.

3. Fractional Distillation Method is used to produce pure oxygen from atmospheric air.

Which of the above statements is/are correct?

a. 1 and 2 only

b. 2 and 3 only

c. 1 and 3 only

d. 1, 2 and 3

Solution: d

Explanation:

Page 69: QUIZ COMPILATIONS – JULY PART 1

GOALTIDE IAS ACADEMY 68

QUIZ COMPILATIONS – JULY PART 1

First statement is correct.

Liquid oxygen has a pale blue colour and is a cryogenic liquid. Cryogenic liquids are

liquefied gases that have a normal boiling point below -238 degrees Fahrenheit (-150

degrees Celsius).

Second statement is correct.

In commerce, liquid oxygen is classified as an industrial gas and is widely used for industrial

and medical purposes. Liquid oxygen is used as an oxidant for liquid fuels in the propellant

systems of missiles and rockets.

Third statement is correct.

How Liquid Medical Oxygen is produced?

There are several methods. The most common production method is separation of oxygen

in what are known as Air Separation Units or ASUs. ASUs are basically plants that separate

large volumes of gases. They use a method called Fractional Distillation Method to

produce pure oxygen from atmospheric air.

1. In this method, gases from the air are separated into various components after

cooling them into a liquid state and then liquid oxygen is extracted from it.

2. Atmospheric air is first cooled to -181°C. Oxygen liquifies at this point. Since, the

boiling point of Nitrogen is -196°C, it remains in a gaseous state. But Argon has a

boiling point similar to that of oxygen (–186°C) and hence a significant amount of

Argon liquifies along with Oxygen.

3. The resultant mixture of Oxygen and Argon is drained, decompressed and passed

through a second low-pressure distillation vessel for further purification.

4. We then get the output as final purified liquid oxygen, which is then transported

using cryogenic containers.

Oxygen can also be produced non-cryogenically, in gaseous form, using selective adsorption. This method leverages the property that under high pressure, gases tend to be attracted to solid surfaces. The higher the pressure, the more the adsorption of gas. If a gas mixture such as air is passed under pressure through a vessel containing an adsorbent bed of ‘zeolite’ that attracts nitrogen more strongly than oxygen, a part or all of the nitrogen will stay in the bed, and the gas exiting the vessel will be richer in oxygen, relative to the mixture entering the vessel.

Q82. Consider the following statements regarding OPEC.

1. It is a permanent, intergovernmental Organization, created at the Baghdad

Conference.

2. OPEC countries contribute more than 60 percent of World’s crude oil reserves.

3. Its headquarters is in Austria, Vienna.

Select the correct code.

Page 70: QUIZ COMPILATIONS – JULY PART 1

GOALTIDE IAS ACADEMY 69

QUIZ COMPILATIONS – JULY PART 1

a. 1 and 2 only

b. 1 and 3 only

c. 3 only

d. 1, 2 and 3

Solution: d

Explanation:

The OPEC Secretariat is the executive organ of the Organization of the Petroleum Exporting

Countries (OPEC). Located in Vienna, it also functions as the Headquarters of the

Organization, in accordance with the provisions of the OPEC Statute.

Qatar terminated its membership on 1 January 2019, currently, the Organization has a total

of 13 Member Countries.

Page 71: QUIZ COMPILATIONS – JULY PART 1

GOALTIDE IAS ACADEMY 70

QUIZ COMPILATIONS – JULY PART 1

Q83. Consider the following statements regarding Shanghai Co-Operation Organization.

1. It is a permanent intergovernmental international organization formed after 2007-

2008 financial crisis.

2. The SCO Secretariat, based in Shanghai, is the main permanent executive body of the

SCO.

3. India, Pakistan, China, Russia and US, all are members of SCO.

Select the correct code.

Page 72: QUIZ COMPILATIONS – JULY PART 1

GOALTIDE IAS ACADEMY 71

QUIZ COMPILATIONS – JULY PART 1

a. 1 only

b. 2 only

c. 3 only

d. None.

Solution: d

Explanation:

First Statement:

Second statement:

Third statement:

Page 73: QUIZ COMPILATIONS – JULY PART 1

GOALTIDE IAS ACADEMY 72

QUIZ COMPILATIONS – JULY PART 1

Q84. Consider the following statements regarding Central Adoption Resource Authority

(CARA).

1. It is a statutory body of Ministry of Women and Children Development under

Juvenile Justice Act, 2015.

2. It is mandated to monitor and regulate in-country and inter-country adoptions of

Indian Children.

3. Government of India has ratified Hague Convention on Inter-country Adoption,

1993.

Which of the above statements is/are correct?

a. 1 and 2 only

b. 2 only

c. 2 and 3 only

d. 1, 2 and 3

Solution: d

Explanation:

Central Adoption Resource Authority (CARA) is a statutory body of Ministry of Women &

Child Development, Government of India. Read below:

Page 74: QUIZ COMPILATIONS – JULY PART 1

GOALTIDE IAS ACADEMY 73

QUIZ COMPILATIONS – JULY PART 1

It functions as the nodal body for adoption of Indian children and is mandated to monitor

and regulate in-country and inter-country adoptions.

CARA is designated as the Central Authority to deal with inter-country adoptions in

accordance with the provisions of the Hague Convention on Inter-country Adoption, 1993,

ratified by Government of India in 2003.

Page 75: QUIZ COMPILATIONS – JULY PART 1

GOALTIDE IAS ACADEMY 74

QUIZ COMPILATIONS – JULY PART 1

CARA primarily deals with adoption of orphan, abandoned and surrendered children

through its associated /recognised adoption agencies.

Q85. Consider the following statements regarding Intergovernmental Science-Policy

Platform on Biodiversity and Ecosystem Services (IPBES).

1. It is United Nation body to strengthen the science-policy interface for biodiversity

and ecosystem services for the conservation and sustainable use of biodiversity.

2. India is a member of IPBES.

Which of the above statements is/are correct?

a. 1 only

b. 2 only

c. Both 1 and 2

d. Neither 1 nor 2

Solution: b

Explanations:

Page 76: QUIZ COMPILATIONS – JULY PART 1

GOALTIDE IAS ACADEMY 75

QUIZ COMPILATIONS – JULY PART 1

India is a member of Intergovernmental Science-Policy Platform on Biodiversity and

Ecosystems since 2012.

Page 77: QUIZ COMPILATIONS – JULY PART 1

GOALTIDE IAS ACADEMY 76

QUIZ COMPILATIONS – JULY PART 1

Link: https://ipbes.net/

Q86. Which of the following languages is/are listed in the Eighth Schedule of the

Constitution of India?

1. Nepali

2. Kashmiri

3. Sanskrit

4. English

5. Manipuri

Select the correct code.

a. 2, 3 and 5 only

b. 1, 2 and 3 only

c. 1, 2, 3 and 5 only

d. 1, 2. 3. 4 and 5

Solution: c

Explanation:

Q87. In the context of Gupta Empire, consider the following statements.

1. By the end of the Gupta period the volume of the Indian trade had increasingly

declined.

2. Gupta period witnessed certain political, social and economic developments which

may be described as the base of feudalism.

3. Forced Labour was practiced during Gupta Empire.

Select the correct code.

a. 1 and 2 only

Page 78: QUIZ COMPILATIONS – JULY PART 1

GOALTIDE IAS ACADEMY 77

QUIZ COMPILATIONS – JULY PART 1

b. 2 and 3 only

c. 1 and 3 only

d. 1, 2 and 3

Solution: d

Explanation:

Statement 1 is correct.

Statement 2 is correct.

Page 79: QUIZ COMPILATIONS – JULY PART 1

GOALTIDE IAS ACADEMY 78

QUIZ COMPILATIONS – JULY PART 1

Statement 3 is correct.

Page 80: QUIZ COMPILATIONS – JULY PART 1

GOALTIDE IAS ACADEMY 79

QUIZ COMPILATIONS – JULY PART 1

Q88. In the context of Early Vedic Period, Brahmavadinis and Sadyodvahas refers to:

a. Educated women groups

b. Saints in early Vedic period

c. Sacred texts to be read by women

d. Sacred placed to read sacred texts

Solution: a

Explanation:

Q89. Consider the following statements regarding Prorogation of Lok Sabha:

1. Lok Sabha is prorogued by the Speaker.

2. House cannot be prorogued while it is in session.

3. Only Lok Sabha can be prorogued.

Which of the above statements are correct?

a. 1 only

b. 2 and 3 only

c. 1 and 3 only

d. None of the above

Solution: d

Explanation:

The presiding officer (Speaker or Chairman) declares the House adjourned sine die, when

the business of a session is completed. Within the next few days, the President issues a

notification for prorogation of the session. However, the President can also prorogue the

House while in session.

Page 81: QUIZ COMPILATIONS – JULY PART 1

GOALTIDE IAS ACADEMY 80

QUIZ COMPILATIONS – JULY PART 1

Q90. Consider the following.

1. Dhaka

2. Kolkata

3. Imphal

Arrange the above from North to South.

a. 2-1-3

b. 3-2-1

c. 3-1-2

d. 1-3-2

Solution: c

Explanation:

The correct sequence is Imphal-Dhaka-Kolkata

Q91. Which of following instruments are used in Indian Parliament for effecting

parliamentary surveillance over administrative action?

1. system of parliamentary Committees

2. Calling Attention

3. discussions on the Motion of Thanks on the President's Address

Page 82: QUIZ COMPILATIONS – JULY PART 1

GOALTIDE IAS ACADEMY 81

QUIZ COMPILATIONS – JULY PART 1

Select the correct code.

a. 1 only

b. 2 and 3 only

c. 1 and 2 only

d. 1, 2 and 3

Solution: d

Explanation:

The various procedural devices like the system of parliamentary Committees; Questions,

Calling Attention, Half-an-Hour Discussion, etc. constitute very potent instruments for

effecting parliamentary surveillance over administrative action.

Significant occasions for review of administration are also provided by the discussions on

the Motion of Thanks on the President's Address, the Budget demands and particular

aspects of governmental policy or situations. These apart, specific matters may be discussed

through motions on matters of urgent public importance, private members' resolutions and

other substantive motions.

Q92. Consider the following statements.

1. Nonylphenol, a hazardous chemical present in detergents, bio-accumulates and can

pose serious environmental and health risks.

2. The detergents contain suspected carcinogens, and ingredients that do not fully

biodegrade.

3. Laundry detergents contains phosphate salts which tends to inhibit the

biodegradation of organic substances.

Which of the above statements is/are correct?

a. 1 and 2 only

b. 2 and 3 only

c. 1 and 3 only

d. 1, 2 and 3

Solution: d

Explanation:

Fist statement is correct.

Nonylphenol, a hazardous chemical present in detergents, is known to enter water bodies

and the food chains. It bio-accumulates and can pose serious environmental and health

risks.

It has been detected in human breast milk, blood and urine, and is associated with

reproductive and developmental effects in rodents. It is recommended to find substitutes of

nonylphenol.

Page 83: QUIZ COMPILATIONS – JULY PART 1

GOALTIDE IAS ACADEMY 82

QUIZ COMPILATIONS – JULY PART 1

Second statement is correct.

The detergents contain suspected carcinogens, and ingredients that do not fully biodegrade.

Third statement is correct:

Many laundry detergents contain approximately 35 to 75 per cent phosphate salts.

Phosphates can cause a variety of water pollution problems. For example, phosphate tends

to inhibit the biodegradation of organic substances. Non-biodegradable substances cannot

be eliminated by public or private wastewater treatment.

Some phosphate-based detergents can also cause eutrophication.

Link: https://www.downtoearth.org.in/blog/water/how-our-detergent-footprint-is-

polluting-aquatic-ecosystems-77935

Q93. Consider the following statements.

1. As per the Constitution, advice tendered by Ministers to the President shall not be

inquired into in any court.

2. As per the Constitution, the Prime Minister shall be appointed by the President elected

unanimously by Council of Ministers.

3. India had women representation in the union council of ministers from the very first

government.

Which of the above statements is/are correct?

a. 1 and 2 only

b. 1 and 3 only

c. 3 only

d. 1, 2 and 3

Solution: b

Explanation:

First statement is correct.

Second statement is incorrect: Elected unanimously part is not mentioned.

Page 84: QUIZ COMPILATIONS – JULY PART 1

GOALTIDE IAS ACADEMY 83

QUIZ COMPILATIONS – JULY PART 1

Third statement is correct:

The first woman cabinet minister was Rajkumari Amrit Kaur of Indian National Congress.

She became the cabinet minister for Health.

Q94. India Industrial Land Bank (IILB) developed by:

a. RBI

b. SEBI

c. Ministry of Finance

d. Department for Promotion of Industry and Internal Trade

Solution: d

Explanation:

Department for Promotion of Industry & Internal Trade (DPIIT) has developed IILB portal,

a GIS-enabled database of industrial areas/clusters across the country to adopt a

committed approach towards resource optimization, industrial upgradation and

sustainability.

The portal serves as a one-stop solution to the free and easy accessibility of all industrial

information including availability of raw material, agriculture, horticulture, minerals, natural

resources; distance from key logistic nodes; layers of terrain and urban infrastructure.

It aims to provide information on available land for prospective investors looking at setting

up units in the Country.

Page 85: QUIZ COMPILATIONS – JULY PART 1

GOALTIDE IAS ACADEMY 84

QUIZ COMPILATIONS – JULY PART 1

It also provides links to State GIS Portals and State Land Banks.

Q95. Consider the following countries.

1. Tajikistan

2. Laos

3. Nepal

4. South Sudan

5. Bolivia

6. Mali

7. Ethiopia

Which of the above is/are landlocked countries?

a. 1, 3, 4, 5 and 6 only

b. 3, 5, 6 and 7 only

c. 1, 2, 3, 4, 5 and 7 only

d. 1, 2, 3, 4, 5, 6 and 7

Solution: d

Explanation:

Today, learn and fix image of all Land-locked in your brain (cerebellum and prefrontal cortex

part of your brain helps you to remember). See the image given below.

Q96. Consider the following statements.

Page 86: QUIZ COMPILATIONS – JULY PART 1

GOALTIDE IAS ACADEMY 85

QUIZ COMPILATIONS – JULY PART 1

1. The president has power to pardon the punishment or sentence of an individual by

a Court Martial.

2. If election of a President is declared void by Supreme Court, all acts of President

before this decision of Supreme Court is declared as invalidated.

Which of the above statements is correct?

a. 1 only

b. 2 only

c. Both 1 and 2

d. Neither 1 nor 2

Solution: a

Explanation:

We will see here article 71 and 72.

Q97. In which of the following states, Sixth Schedule of India Constitution is not applied?

a. Mizoram

b. Tripura

c. Manipur

d. Meghalaya

Solution: c

Explanation:

Idhar for this question who all have done wrong, just comment. I will book a special

appointment.

States which are included in sixth schedule are:

Assam, Meghalaya, Tripura and Mizoram

Page 87: QUIZ COMPILATIONS – JULY PART 1

GOALTIDE IAS ACADEMY 86

QUIZ COMPILATIONS – JULY PART 1

Q98. Consider the following statements.

1. Gandhiji became present of Indian National Congress in Belgaum session.

2. Formation of Swaraj Party

3. Passage of Poorna Swaraj Resolution

4. Nehru Report

Arrange the above events of Indian National movement in sequence.

a. 2-1-4-3

b. 1-2-4-3

c. 2-1-3-4

d. 2-4-1-3

Solution: a

The Swaraj Party was established as the Congress-Khilafat Swaraj Party. It was a political

party formed in India in January 1923 after the Gaya annual conference in December 1922

of the National Congress after Gandhi called off Non-cooperation Movement following

Chauri-Chaura incident.

Mahatma Gandhi became President of Indian National Congress at Belgaum session in

1924.

Nehru Report came in 1928.

The Indian National Congress, on 19 December 1929, passed the historic 'Poorna Swaraj'

(total independence) resolution at its Lahore session.

Q99. Amir Khushru is related to which style of singing in Hindustani music? a. Dhrupad. b. Thumri. c. Khayal. d. Ragasagar.

Solution: c Explanations: The word ‘Khayal’ is derived from Persian and means “idea” or “imagination”. The origin of this style was attributed to Amir Khushru. A Khayal composition is also known as Bandish. Q100. Consider the following events of Indian National movement.

1. Champaran Satyagraha

2. August Declaration.

3. Lucknow Pact.

4. Home Rule Movement of Tilak.

Arrange the following events in chronological order.

Page 88: QUIZ COMPILATIONS – JULY PART 1

GOALTIDE IAS ACADEMY 87

QUIZ COMPILATIONS – JULY PART 1

a. 4-3-1-2

b. 3-4-2-1

c. 4-3-2-1

d. 3-4-1-2

Solution: a

Explanation:

Home Rule Movement of Tilak – April 1916.

Lucknow Pact- December 1916.

Champaran Satyagraha- April 1917.

August Declaration- August 1917.

Q101. Consider the following Pairs.

Places in news Location

1. Turkana region South Sudan

2. Haiti Pacific Ocean

Which of the above pairs is/are correct?

a. 1 only

b. 2 only

c. Both 1 and 2

d. Neither 1 nor 2

Solution: d

Explanation:

Places in news Location

Turkana region Kenya

Haiti Caribbean Sea

Page 89: QUIZ COMPILATIONS – JULY PART 1

GOALTIDE IAS ACADEMY 88

QUIZ COMPILATIONS – JULY PART 1

Q102. Consider the following statements.

1. Global public debt rose to an all-time high as the pandemic-induced decline in

government revenues and increased spending to support growth-oriented policies.

2. Growth in Bank deposits slows down while the growth in credit increases during

pandemic (2020-21) in India.

Which of the above statements is/are correct?

a. 1 only

b. 2 only

c. Both 1 and 2

d. Neither 1 nor 2

Solution: a

Explanation:

The RBI recently released the 23rd edition of the biannual Financial Stability Report (FSR) for

July 2021. These two statements have been taken from it. We will cover all major trends

gradually in Daily Quiz.

First statement is correct.

Page 90: QUIZ COMPILATIONS – JULY PART 1

GOALTIDE IAS ACADEMY 89

QUIZ COMPILATIONS – JULY PART 1

Second statement is incorrect:

2020-21 recorded the lowest credit growth in four years while aggregate deposits

increased.

Current account and savings account (CASA) deposits grew at a faster pace than term

deposits, possibly reflecting the propensity of savers to hold more liquid assets in the highly

uncertain pandemic situation.

Q103. Which of the following can enhance the “Precision Farming” in India?

1. Use artificial intelligence (AI), Global Navigation Satellite System (GNSS), and Big

Data.

2. Use of colour chart and GreenSeeker (a handheld sensor) to determine the nitrogen

requirement for rice and maize crops

3. Liberalised geospatial sector

4. Increasing subsidies on Fertilizers

Select the correct code.

a. 1 and 2 only

b. 1, 2 and 3 only

c. 2 and 4 only

d. 1, 2, 3 and 4

Solution: b

Explanation:

Page 91: QUIZ COMPILATIONS – JULY PART 1

GOALTIDE IAS ACADEMY 90

QUIZ COMPILATIONS – JULY PART 1

Precision farming is an approach where inputs are utilized in precise amounts to get

increased average yields, compared to traditional cultivation techniques.

First and second statement are correct.

Precision farming combines the power of artificial intelligence (AI), Global Navigation

Satellite System (GNSS), and Big Data. It is efficient, climate-smart, and nutrition-sensitive.

Citing an ongoing project by the ICAR-Indian Agricultural Research Institute, Raj Singh, head

of the institute’s agronomy division, said, “Using readily available devices in the market like

colour chart and GreenSeeker (a handheld sensor), we are trying to determine the

nitrogen requirement for rice and maize crops. This precision nitrogen management will

help save the fertilizer costs, prevent environmental harm and lead to efficient use of the

huge nitrogen imports by the Centre.

Third statement is correct:

Fourth is incorrect:

More subsidies, more misuse of fertilizers, that is not the objective of Precision farming.

Q104. Consider the following statements with respect to Rajya Sabha.

1. If Prime minister is the member of House, he will be declared as a leader of the House.

2. If Prime Minister is not the Leader of the House, then Chairman appoints leader from

House.

3. The ‘Leader of the House’ is defined in the Constitution.

Which of the above statements is/are correct?

a. 1 only

b. 2 and 3 only

c. 1 and 3 only

d. None

Page 92: QUIZ COMPILATIONS – JULY PART 1

GOALTIDE IAS ACADEMY 91

QUIZ COMPILATIONS – JULY PART 1

Solution: a

Explanation:

Rule 2(1) of the Rules of Procedure and Conduct of Business in the Council of States (Rajya

Sabha) defines the Leader of Rajya Sabha as follows: Third statement is incorrect.

"Leader of the Council" means the Prime Minister, if he is a member of the Council, or a

Minister who is a member of the Council and is nominated by the Prime Minister to

function as the Leader of the Council. First statement is correct.

Second statement is incorrect.

Q105. ‘Social Climate Fund’ was recently announced by:

a. UNEP

b. UNFCCC Secretariat

c. European Union

d. WTO

Solution: c

Explanation:

Q106. With regard to the judge of a High Court, consider the following statements:

Page 93: QUIZ COMPILATIONS – JULY PART 1

GOALTIDE IAS ACADEMY 92

QUIZ COMPILATIONS – JULY PART 1

1. He can be removed from his office by the Governor on the recommendation of the

President.

2. He can resign from his office by writing to the President.

Which of the statements given above is/are correct?

a. 1 only

b. 2 only

c. Both 1 and 2

d. Neither 1 nor 2

Solution: b

We will here refer to article 217:

He can be removed from his office by the President on the recommendation of the

Parliament. Although, the High Court judge does not work on the pleasure of the President.

In article mentioned above, it is written under provision of Article 124.

We will also see here, what is written in article 124 (4).

Article 124(4) of the Constitution of India states: “A Judge of the Supreme Court shall not be

removed from his office except by an order of the President passed after an address by each

House of Parliament supported by a majority of the total membership of that House and by a

majority of not less than two-thirds of the members of that House present and voting has

been presented to the President in the same session for such removal on the ground of

proved misbehavior or incapacity.”

He can resign from his office by writing to the President. Hence, only statement 2 is

correct.

Q107. Consider the following statements regarding Indian Renewable Energy Development

Agency Limited (IREDA).

1. It is a Private Limited Government Company established as a Non-Banking Financial

Institution.

Page 94: QUIZ COMPILATIONS – JULY PART 1

GOALTIDE IAS ACADEMY 93

QUIZ COMPILATIONS – JULY PART 1

2. IREDA has been notified as a “Public Financial Institution” under Companies Act,

1956.

3. It aims to maintain its position as a leading organisation to provide efficient and

effective financing in renewable energy and conservation projects.

Select the correct code.

a. 1 and 2 only

b. 2 and 3 only

c. 1 and 3 only

d. 1, 2 and 3

Solution: b

Explanation:

First statement is incorrect. It is Public, not Private.

Q108. The Chairperson of National Human Rights Commission:

Page 95: QUIZ COMPILATIONS – JULY PART 1

GOALTIDE IAS ACADEMY 94

QUIZ COMPILATIONS – JULY PART 1

a. Chief Justice of Supreme Court

b. Judge of Supreme Court

c. Prime Minister

d. Both a and b

Solution: d

Explanation:

Why we asked this question because of The Protection of Human Rights (Amendment) Bill,

2019 which made several changes in the Act of 1993.

Under the earlier Act, the chairperson of the NHRC is a person who has been a Chief Justice

of the Supreme Court. The Bill amends this to provide that a person who has been Chief

Justice of the Supreme Court, or a Judge of the Supreme Court will be the chairperson of

the NHRC.

Q109. Ulatbansi is a language of poetry. Poems written in it follow a style in which everyday

meanings are inverted. Ulatbansi is a distinct contribution of which Bhakti poet?

a. Surdas

b. Mirabai

c. Kabir

d. Tukaram

Solution: c

Explanation:

Kabir’s poems have survived in several languages and dialects; and some are composed in

the special language like Ulatbansi (upside-down sayings), written in a form in which

everyday meanings are inverted. These hint at the difficulties of capturing the nature of the

Page 96: QUIZ COMPILATIONS – JULY PART 1

GOALTIDE IAS ACADEMY 95

QUIZ COMPILATIONS – JULY PART 1

Ultimate Reality in words: expressions such as ‘the lotus which blooms without flower’ or

the ‘fire raging in the ocean’ convey a sense of Kabir’s mystical experiences.

Q110. Consider the following Arab nations.

1. Saudi Arabia

2. Yemen

3. Oman

4. UAE

Which of the above nations does Tropic of Cancer Passes?

a. 1, 2 and 4 only

b. 1 and 4 only

c. 2, 3 and 4 only

d. 1, 3 and 4 only

Solution: D

Tropic of Cancer passes through Saudi Arabia, UAE and Oman (for middle east nations).